diff options
author | Norbert Preining <norbert@preining.info> | 2019-09-02 13:46:59 +0900 |
---|---|---|
committer | Norbert Preining <norbert@preining.info> | 2019-09-02 13:46:59 +0900 |
commit | e0c6872cf40896c7be36b11dcc744620f10adf1d (patch) | |
tree | 60335e10d2f4354b0674ec22d7b53f0f8abee672 /info/digests/texhax/97 |
Initial commit
Diffstat (limited to 'info/digests/texhax/97')
-rw-r--r-- | info/digests/texhax/97/texhax.01 | 752 | ||||
-rw-r--r-- | info/digests/texhax/97/texhax.02 | 410 | ||||
-rw-r--r-- | info/digests/texhax/97/texhax.03 | 215 | ||||
-rw-r--r-- | info/digests/texhax/97/texhax.04 | 337 | ||||
-rw-r--r-- | info/digests/texhax/97/texhax.05 | 434 | ||||
-rw-r--r-- | info/digests/texhax/97/texhax.06 | 442 | ||||
-rw-r--r-- | info/digests/texhax/97/texhax.07 | 232 | ||||
-rw-r--r-- | info/digests/texhax/97/texhax.08 | 258 | ||||
-rw-r--r-- | info/digests/texhax/97/texhax.09 | 431 | ||||
-rw-r--r-- | info/digests/texhax/97/texhax.10 | 168 | ||||
-rw-r--r-- | info/digests/texhax/97/texhax.11 | 366 |
11 files changed, 4045 insertions, 0 deletions
diff --git a/info/digests/texhax/97/texhax.01 b/info/digests/texhax/97/texhax.01 new file mode 100644 index 0000000000..3d1db25396 --- /dev/null +++ b/info/digests/texhax/97/texhax.01 @@ -0,0 +1,752 @@ +(Message texhax/v97:1) +From texhax-digest-owner@nottingham.ac.uk Sat Feb 8 19:30:27 1997 +Received: from nottingham.ac.uk (jess.ccc.nottingham.ac.uk [128.243.40.193]) by granby.ccc.nottingham.ac.uk (8.6.12/8.6.12) with SMTP id TAA18668 for <cczdao@unix.ccc.nottingham.ac.uk>; Sat, 8 Feb 1997 19:30:26 GMT +Received: from majordom by nottingham.ac.uk with local (Exim 1.58 #10) + id 0vtITZ-0007bc-00; Sat, 8 Feb 1997 19:30:17 +0000 +From: owner-texhax-digest@nottingham.ac.uk +To: texhax-digest@nottingham.ac.uk +Subject: TeXhax Digest V1997 #1 +Reply-To: TeXhax@tex.ac.uk +Precedence: bulk +Message-Id: <E0vtITZ-0007bc-00@nottingham.ac.uk> +Date: Sat, 8 Feb 1997 19:30:17 +0000 + + +TeXhax Digest Saturday, 8 February 1997 Volume 1997 : Number 001 + +(incorporating UKTeX Digest) + +Today's Topics: + + Re> TeXhax digest + Testing if string starts with particular letter or word + Gothic Letters in LaTeX + Where can I find the psboxit package? + TeX directory structure (tds) 0.9993 available + web2c 7.0 and friends available + +---------------------------------------------------------------------- + +From: KNAPPEN@VKPMZD.kph.Uni-Mainz.DE +Date: Fri, 20 Dec 1996 19:09:17 +0100 +Subject: Re> TeXhax digest + +Colin Maison asked about: + + \font\titlefont=cmssdc10 + \ifx\titlefont\nullfont + \wlog{Using \string\nullfont}% + \fi + \bye + +This one is a classic pitfall. There sould be a \relax at the end of the +first line. Without that \relax, TeX scanns ahead for the keywords `at' +or `scaled' which are permissible in a font definition. Because \if +is expandable, the \if is expanded *before* the font can be assigned, thus +it is still the \nullfont at that moment. + +- --J"org Knappen. + + + +------------------------------ + +From: David Rhead <cczdgr@unix.ccc.nottingham.ac.uk> +Date: Sun, 22 Dec 1996 16:15:37 +0000 (GMT) +Subject: Testing if string starts with particular letter or word + +We have a database of information about the modules from which students' +courses are constructed. We typeset a `Catalogue of Modules' by +`publishing from the database'. I'm wondering about varying the detail +given in the catalogue depending on whether (1) a module is restricted to a +particular group of students, or (2) may have spare places. + +The database has a text field called `target students'. The `house style' +is that, in case (1), the text starts with + Restricted to students registered for ... +but, in case (2), the text starts with something else. + +Currently the `database to .tex file' software gives a file that starts +with + \usepackage{catalogue} +and continues with, for each module, commands of the form + % Set \TargetStudents to the text from the database field + \renewcommand{\TargetStudents}{...} + % More \renewcommands for other stuff from the database + ... + % Typeset the catalogue entry from the information held in + % \TargetStudents, etc. + \ProcessModule + +I'd like to write a package file catalogue.sty (or maybe a class file +catalogue.cls) that defines a \ProcessModule containing a fragment +something like + \if\beginswith{\TargetStudents}{Restricted} + % Text held in \TargetStudents starts with `Restricted'. + % We just typeset a skeleton entry in the catalogue. + ... + \else + % Typeset a full entry in the catalogue + ... + \fi +where \beginswith is a command that I'm having difficulty finding or +writing. + +Essentially I want something that, when given a string of text as `the +command' \TargetStudents, tests whether the string starts with a particular +substring. Although, it would be nice to be able to test whether the +string starts with `Restricted', it would actually be sufficient to +test whether it starts with the letter R. + +I've tried to use LaTeX's + \if@nextchar R +in catalogue.sty, but I didn't seem able to get it to do what I want. +(Perhaps the approach could work, but I got the details wrong?) + +Can anyone suggest an approach (or code-fragment) that is suitable for this +situation? E.g., is there any software in CTAN that involves testing +whether a string held in `a command' starts with a particular letter or +string? (If there's a fragment of code somewhere, I could presumably +imitate it.) + + +David Rhead +david.rhead@nottingham.ac.uk + + + +------------------------------ + +From: Vadim Zelenkov <zelenkov@gray.isir.minsk.by> +Date: Tue, 14 Jan 1997 17:01:08 +-200 +Subject: Gothic Letters in LaTeX + +Dear Sirs, + +Is it possible to install some gothic letters in LaTeX text? This +problem is extremely improtant for our mathematicians... + +Thank you! + +Vadim Zelenkov, +Head, Laboratory of New Educational Technologies, +International Sakharov Institute of Radioecology, +Minsk, Belarus + + + + +------------------------------ + +From: David.Rhead@vme.ccc.nottingham.ac.uk +Date: Thu,16 Jan 97 17:19:56 GMT +Subject: Where can I find the psboxit package? + +psboxit -- as mentioned on page 330 of the Companion -- seems just what I want. + +Unfortunately, I can't find it at ftp.tex.ac.uk. + +Is it in CTAN in a non-obvious place? + +If not, does anyone know where I can get it from? + + David Rhead + +------------------------------ + +From: "K. Berry" <kb@cs.umb.edu> +Date: Fri, 7 Feb 1997 16:57:57 -0500 (EST) +Subject: TeX directory structure (tds) 0.9993 available + +Another draft of the TeX Directory Structure (TDS) standards document +is now available from: + + http://www.tug.org/tds/ + ftp://ftp.tug.org/tex/tds.tar.gz + ftp://ftp.tug.org/tex/tds/ + +If ftp.tug.org is busy, try ftp.cs.umb.edu or tug2.cs.umb.edu. Soon it +will be on CTAN as well, in the `tds' directory (list of CTAN hosts appended). + +The main change from the last released draft (published in TUGboat) is +to combine the directories for the various whatever-to-pk converters +(gsftopk, ps2pk, ...) into one directory `modeless', instead of having +one directory per application. This simplifies search path specifications. + +Please send comments to twg-tds@mail.tug.org. + +kb@mail.tug.org, for the TDS committee + + +prompt$ finger ctan@ftp.tug.org +[tug.cs.umb.edu] +... +Plan: +In order to reduce network load, it is recommended that you use the +Comprehensive TeX Archive Network (CTAN) host which is located in the +closest network proximity to your site. Alternatively, you may wish to +obtain a copy of the CTAN via CD-ROM (see help/CTAN.cdrom for details). + +Known mirrors of the CTAN reside on (alphabetically): + cis.utovrm.it (Italia) /TeX + ctan.unsw.edu.au (NSW, Australia) /tex-archive + dongpo.math.ncu.edu.tw (Taiwan) /tex-archive + ftp.center.osaka-u.ac.jp (Japan) /CTAN + ftp.ccu.edu.tw (Taiwan) /pub/tex + ftp.cdrom.com (West coast, USA) /pub/tex/ctan + ftp.comp.hkbu.edu.hk (Hong Kong) /pub/TeX/CTAN + ftp.cs.rmit.edu.au (Australia) /tex-archive + ftp.cs.ruu.nl (The Netherlands) /pub/tex-archive + ftp.cstug.cz (The Czech Republic) /pub/tex/CTAN + ftp.duke.edu (North Carolina, USA) /tex-archive + ftp.ee.up.ac.za (South Africa) /tex-archive + ftp.funet.fi (Finland) /pub/TeX/CTAN + ftp.gwdg.de (Deutschland) /pub/dante + ftp.jussieu.fr (France) /pub4/TeX/CTAN + ftp.loria.fr (France) /pub/unix/tex/ctan + ftp.mpi-sb.mpg.de (Deutschland) /pub/tex/mirror/ftp.dante.de + ftp.nada.kth.se (Sweden) /pub/tex/ctan-mirror + ftp.rediris.es (Espa\~na) /mirror/tex-archive + ftp.rge.com (New York, USA) /pub/tex + ftp.riken.go.jp (Japan) /pub/tex-archive + ftp.tu-chemnitz.de (Deutschland) /pub/tex + ftp.u-aizu.ac.jp (Japan) /pub/tex/CTAN + ftp.uni-augsburg.de (Deutschland) /tex-archive + ftp.uni-bielefeld.de (Deutschland) /pub/tex + ftp.unina.it (Italia) /pub/TeX + ftp.uni-stuttgart.de (Deutschland) /tex-archive (/pub/tex) + ftp.univie.ac.at (\"Osterreich) /packages/tex + ftp.ut.ee (Estonia) /tex-archive + ftpserver.nus.sg (Singapore) /pub/zi/TeX + kadri.ut.ee (Estonia) /pub/tex + src.doc.ic.ac.uk (England) /packages/tex/uk-tex + sunsite.cnlab-switch.ch (Switzerland) /mirror/tex + sunsite.icm.edu.pl (Poland) /pub/CTAN + sunsite.queensu.ca (Canada) /pub/tex-archive + sunsite.unc.edu (North Carolina, USA) /pub/packages/TeX + wuarchive.wustl.edu (Missouri, USA) /packages/TeX + +Known partial mirrors of the CTAN reside on (alphabetically): + ftp.adfa.oz.au (Australia) /pub/tex/ctan + ftp.fcu.edu.tw (Taiwan) /pub2/tex + ftp.germany.eu.net (Deutschland) /pub/packages/TeX + ftp.jaist.ac.jp /pub/TeX/tex-archive + ftp.uu.net (Virginia, USA) /pub/text-processing/TeX + nic.switch.ch (Switzerland) /mirror/tex + sunsite.dsi.unimi.it (Italia) /pub/TeX + sunsite.snu.ac.kr (Korea) /shortcut/CTAN + +Please send updates to this list to <ctan@urz.uni-heidelberg.de>. + +The participating hosts in the Comprehensive TeX Archive Network are: + ftp.dante.de (Deutschland) + -- anonymous ftp /tex-archive (/pub/tex /pub/archive) + -- gopher on node gopher.dante.de + -- e-mail via ftpmail@dante.de + -- World Wide Web access on www.dante.de + -- Administrator: <ftpmaint@dante.de> + + ftp.tex.ac.uk (England) + -- anonymous ftp /tex-archive (/pub/tex /pub/archive) + -- gopher on node gopher.tex.ac.uk + -- NFS mountable from nfs.tex.ac.uk:/public/ctan/tex-archive + -- World Wide Web access on www.tex.ac.uk + -- Administrator: <ctan-uk@tex.ac.uk> + + + +------------------------------ + +From: kb@cs.umb.edu +Date: Sat, 8 Feb 1997 14:28:06 -0500 (EST) +Subject: web2c 7.0 and friends available + +I've released version 7.0 of Web2c, a port of the basic TeX project +programs (TeX, Metafont, MetaPost, GFtoPK, etc.) to Unix. I've also +released new versions of my modified drivers (dvipsk, xdvik, dviljk). + +You can get it via http://www.tug.org/web2c/, or by ftp from: + + (Boston) ftp://ftp.cs.umb.edu/pub/tex/web2c/texk.tar.gz (programs) + ftp://ftp.cs.umb.edu/pub/tex/web2c/texmflib.tar.gz (minimal lib) + +texk.tar.gz is the combined archive; you can also get them separately: +{dviljk,dvipsk,web2c,web,xdvik}.tar.gz. + +(Also on tug.cs.umb.edu and tug2.cs.umb.edu, same directory; + ftp.cs.umb.edu is an old and slow machine.) + +Soon they will be available from the CTAN sites and their mirrors: + + (Germany) ftp://ftp.dante.de/tex-archive/systems/web2c + (England) ftp://ftp.tex.ac.uk/tex-archive/systems/web2c + +See the ftp retrieval instructions below for details and all the mirrors. + +Instructions for sending bug reports (and joining the mailing list) are +in the distribution file kpathsea/BUGS. + +I've tried to record the names of all the many people have sent bug +reports, suggestions, and/or patches in the ChangeLog's. Thank you all. +I'd also like to thank all the long-suffering pretesters, and three +people in particular -- + Thomas Esser, for teTeX and so much else; + Ulrik Vieth, for the MetaPost port, doc improvements, and so much else; + Wayne Sullivan, for the dynamic memory and so much ...; +Thanks also to Rick Martin and the UMass/Boston Math & Computer Science + Dept. for the computer and network facilities. +And let's not forget Don Knuth, http://www-cs-faculty.Stanford.EDU/~knuth/, + who started it all. +(Also thanks to Glenn Gould, Jane Siberry, Leonard Cohen, Donald Westlake, + Laurie Anderson, ee cummings, Stephen King, Johann Sebastian Bach, + and the seals and birds on the not-so-wild shore just outside the house. :-) + + +kb@cs.umb.edu +Member of the League for Programming Freedom -- write lpf@uunet.uu.net. + + +A summary of the changes follows; big changes are summarized in few words, +so read carefully. Read the manuals, too. Please. + +kpathsea 3.0 +* Distribution terms changed to the GNU Library General Public License. +* Default directory structure matches the TDS standard. The most + important change is in the fonts/ directory: the file type (tfm, pk) + is now directly underneath fonts/. Also, mft/ is now at the top level. + `modeless' is used for gsftopk/ps2pk/etc.-generated fonts. +* Change in searching algorithm: if the name already has a suffix + (foo.sty), search for it by itself first, before appending the default + suffix (foo.sty.tex). This loses if you have an executable named + foo.bar, with documentation in foo.bar.tex (foo.bar will be found + first). But this is unusual, and the other case is common. +* Environment variables PATH_program (e.g., TEXINPUTS_latex) checked + for, as with configuration file values (TEXINPUTS.latex). +* Multiple TEXMF trees supported, via the following new facility: +* Shell brace expansion {foo,bar} implemented, thanks to Bash. +* Multiple ls-R files supported via the TEXMFDBS path. +* An ``alias'' database for files in ls-R supported, to handle 8.3 problems. +* Directories in the ls-R file can be relative (./) as well as absolute (/). +* New MakeTeX{PK,TFM,MF} scripts with more features, based on teTeX's. +* DVIPSMAKEPK (etc.) actually overrides the name `MakeTeXPK', as documented. +* MakeTeX* can be globally disabled via configure options. +* Support for building as a shared library with configure --enable-shared. +* Directories whose names begin with `.' are ignored. +* Some warnings can be suppressed by TEX_HUSH cnf value or envvar. +* kpsewhich has many new capabilities, documented via --help and in the manual. +* texfonts.map parsing has an include directive, and @c is a comment. +* Input lines in texmf.cnf can be continued with \. +* TEXMFLOG can be set in texmf.cnf, as previously documented. +* SELFAUTODIR and SELFAUTOPARENT are defined according to the location + of the binary, and used to search for texmf.cnf. +* /// (or more) is equivalent to //. +* If $HOME = / (as with root on some systems), avoid creating // in paths. +* Support for AFM, PostScript Type 1 font, source, documentation, Omega, + and MetaPost-related file formats. +* New header kpathsea.h with all #include files. +* Under DOS, ls-R lines need not start with . or / or \ (e.g., c:). +* Amiga support. + +web2c 7.0 +* Update from Knuth for tex, mf, dvitype, pltotf, vptovf, and tangle, + John Hobby's MetaPost (and support programs) included, + update from P. Breitenlohner to dvicopy 1.5 and patgen 2.3. +* Update for kpathsea 3.0. +* Texinfo manual written. +* Amiga, OS/2, Windows NT support in the sources. +* TeX: + - if -shell-escape or shell_escape in texmf.cnf is enabled, + \write18{str} passes str to system(3) for processing by sh. + - MLTeX extensions (\charsubdef, etc.) can be enabled at runtime. + - Many array sizes can be set dynamically, in texmf.cnf. + - MakeTeXTFM installed and enabled by default. + - IPC communications to an external program for incremental DVI reading. + - texfonts.map applies to TFM files, as originally intended. + - More than 256 fonts are allowed. + - DVI comment (with timestamp) can be overridden. + - tex.web's consistency checks on .fmt files restored. +* Metafont: + - A few array sizes can be set dynamically, in texmf.cnf. + - X11 support disabled by default; configure --with-x to enable it. + - MakeTeXMF enabled and installed by default. +* TeX, Metafont, MetaPost: + - If the first line of the main input file starts with %&, the rest of + the line is used as the fmt/base/mem file (if it can be found), + except that %&ini means initex/inimf/inimpost. + - One binary: initex/mf/mpost capabilities merged with virtex/mf/mpost. + - New option -progname sets program name independently of argv[0]. + - Eight-bit filenames are output correctly in ^^ notation, instead of + always being simplified to ^^@. + - MakeTeX* scripts can be named and disabled via texmf.cnf. + - MakeTeX* scripts are silent if \batchmode is in effect. + - TEXMFINI environment variable checked for pool/fmt/base/mem lookups. + - \openout (TeX) and write (MP) by default will not write to files + whose names start with `.'. + - ``Small'' versions no longer supported without source hacking. + - Length bug regarding single non-ASCII characters in MF and MP fixed. +* BibTeX: + - Some arrays dynamically reallocated as needed, other sizes increased. + - Default behavior is verbose; new -terse option changes that. + - Statistics are written to the .blg file. + - Does not look at TEXINPUTS to find .bst files, but does check + TEXBIB (after BIBINPUTS). + - Does not interactively ask for the top-level .aux name. +* All programs: + - Supply default extensions and default output files. + - Support the standard --help and --version options. + - Filenames not arbitrarily restricted in length. + - Output written in the current directory, not the directory of + an input file. (Most interesting for tangle.) + - Times-Roman etc. work as font names, via texfonts.map and subsidiaries. +* dvitype prints decimal opcode numbers with --show-opcodes. +* bibtex and gftodvi never interactively ask for filenames; + dvitype and dvicopy do not enter interactive dialogues. +* mft looks in MFTINPUTS, not TEXINPUTS. +* Configuration: + - Optional features are enabled via configure --with-* and --enable-* + options. Run configure --help. + - There is no small TeX, Metafont, or MetaPost. If you want it, you'll + have to change the change files, #define SMALL* yourself, and do + whatever else is necessary yourself. +* Source hierarchy substantially simplified. +* Web2c: + - Support for ifndef...endifn, a la ifdef. + - Support for const expressions. + - Can generate ANSI code. + +dvipsk 5.66a +* Update for kpathsea 3.0 and dvips 5.66. +* No default config.ps is installed. +* Partial downloading of Type 1 fonts supported via -j (j in config file). +* Compressed headers and figures (names ending in .Z or .gz) are + automatically uncompressed. +* Fonts and macros now distributed separately on CTAN. +* dvihps merged in, i.e., hyperdvi html: specials supported. +* Manual restructured and new text regarding PostScript fonts added. +* .dvipsrc is looked for along a path, and the DVIPSRC value overrides. +* -Pfoo does not require config.foo; and if in fact config.foo doesn't exist, + the default output is to | lpr -Pfoo. +* -o - outputs to standard output. +* -E does not require single-page input documents. +* -d -1 does not include the size of every memory allocation; + new information about dvips config file reading is included. +* W messages from config.ps are omitted if -q is specified. +* -E turns off -N and -K. +* Improve checksums and ligkern table output in Afm2tfm. +* Small improvements to epsf.tex. + +xdvik 20a +* Update for kpathsea 3.0. +* xhdvi merged in, i.e., hyperdvi html: specials supported via libwww. +* gsftopkk included. +* Adjustable rulers on magnifiers, and grids on the whole page. +* Many function keys supported. +* Online help via H/Help/etc. while the program is running. +* Shrink button factors determined at runtime. +* Exit with bad status via Abort button, Q keystroke. +* Update for xdvi patchlevel 20 and gsftopk 1.12, implying the following: +* X resource name for the Metafont mode now `mfMode'. +* MakeTeXPK option/resource now `makepk'. +* Compressed (.Z or .gz) header and figure files automatically uncompressed, + and shell escape specials ("`...) turned off by default. +* Decompression does not happen with -nopostscript, etc. +* New options -safer, -interpreter, -nogssafer, -gspalette. + +dviljk 2.6 +* Update for kpathsea 3.0. +* Update for dvi2xx 0.51-13. +* dvihp script handles options a la dvips, and is installed by default. +* Raster characters and fonts can be downloaded compressed. +* - option (operate as pipe) works again. +* More useful debugging output. +* Bug in page-skipping fixed. +* But in printing character 0 fixed. +* Fonts rearranged per TDS, and `q' changed to `8t'. + + +Long-winded instructions: + +`unixtex.ftp': Obtaining TeX +============================ + + This is `ftp://ftp.tug.org/tex/unixtex.ftp', last updated 7 February +1997. Also available as `http://www.tug.org/unixtex.ftp'. The IP +address is currently [158.121.106.10], and the canonical host name is +currently `tug.cs.umb.edu'. It is also in Kpathsea source +distributions as `etc/unixtex.ftp' (although the network version is +usually newer). Mail <kb@mail.tug.org> with comments or questions. + + Following are general instructions for Unix or other sites who wish to +acquire the Web2c distribution, (plain) TeX, LaTeX (2e), BibTeX, +Metafont, MetaPost, DVI processors for the X window system, PostScript, +the PCL language in the HP LaserJet, and related programs. They are +oriented towards building from the original sources, though some +information on alternative packages is included in the last section. +See also `http://www.tug.org/web2c', the Web2c and Kpathsea home page. + + Please consider joining the TeX Users Group (TUG) to help support the +maintenance and development of the programs you retrieve. Email +<tug@tug.org> or see `http://www.tug.org' for information and a +membership form. + + For actual installation instructions after obtaining the necessary +sources, *note Installation::.. A copy is in the distribution file +`kpathsea/INSTALL'. + +Electronic distribution +- ----------------------- + + In many places we refer to CTAN:. This is *both* a host name and a +directory name. Here are some primary locations: + + `ftp://ftp.cdrom.com/pub/tex/ctan' (California, USA) + `ftp://ftp.dante.de/tex-archive' (Germany) + `ftp://ftp.tex.ac.uk/tex-archive' (England) + +CTAN has many mirrors worldwide; see the top-level file +`README.mirrors' from one of the sites above, or finger +<ctan@ftp.tug.org>, or see `http://www.tug.org/CTAN.sites'. A list +current as of the time of distribution is in the top-level file +`./MIRROR'. + + You can also access CTAN via the World Wide Web, Gopher, electronic +mail, or NFS. The same `README.mirrors' file explains how. + + You will need to retrieve some or all of the following archives, +depending on your needs (don't forget to set binary mode for file +transfers): + +`CTAN:/systems/web2c/texmflib.tar.gz' + A basic collection of fonts (TFM files only) and macro packages + (including Texinfo and LaTeX 2e). It unpacks into `texmf/'; if + you change the structure of this hierarchy, you will also have to + change the default search paths (*note Changing search paths::.). + It is required unless you already have these files, in which case + you should change the default paths as necessary to find them. + +`CTAN:/systems/web2c/web.tar.gz' + The original WEB source files, written mostly by Don Knuth. + Required unless you already have this `web' version. (The WEB + sources change irregularly with respect to Web2c itself.) Unpacks + into `web2c-VERSION'. + +`CTAN:/systems/web2c/web2c.tar.gz' + The Web2c system. Required. Also unpacks into `web2c-VERSION'. + +`CTAN:/dviware/dvipsk.tar.gz' + DVI-to-PostScript translator. Unpacks into `dvipsk-VERSION'. + Optional. + +`CTAN:/dviware/xdvik.tar.gz' + X window system DVI previewer. Unpacks into `xdvik-VERSION'. + Optional. + +`CTAN:/dviware/dviljk.tar.gz' + DVI-to-PCL (HP LaserJet) translator. Unpacks into + `dviljk-VERSION'. Optional. + + All that said, the originating host for the software above is +`ftp.tug.org'. You can retrieve these distributions (but not much +else) from the `tex/' directory on that host. + +CD-ROM distribution +- ------------------- + + Numerous organizations distribute various TeX CD-ROM's: + + * TUG, UK TUG, and GUTenberg (French-speaking TeX user group) + collaborated to produce the `TeX Live' CD-ROM, based on teTeX, + which in turn is based on Web2c; email <tex-live@mail.tug.org> or + see `http://www.tug.org/tex-live.html'. + + * NTG (Dutch-speaking TeX user group) produced the 4allTeX CD-ROM; + email <ntg@nic.surfnet.nl>, or see + `http://ei0.ei.ele.tue.nl/ntg/'. This is a runnable system. + + * The Free Software Foundation's `Source Code CD-ROM' contains the + minimal TeX source distribution described in the previous section + (i.e., enough to print GNU documentation); email + <gnu@prep.ai.mit.edu> or finger <fsf@prep.ai.mit.edu>. + + * The Gateway! CD-ROM set contains a runnable NetBSD/Amiga + distribution and sources; see + `http://www.netbsd.org/Sites/cdroms.html'. + + * The Prime Time TeXcetera CD-ROM is a copy of CTAN (see the previous + section); email <ptf@ctcl.com> or see `http://www.ptf.com/ptf/'. + + * Walnut Creek's TeX CD-ROM is also a copy of CTAN; email + <info@cdrom.com> or see `http://www.cdrom.com:/titles/tex.html'. + + * Most Linux distributions include some TeX package based on Web2c; + see the Linux documentation file `Distribution-HOWTO' for a + comparison of Linux distributions, available (for example) via + `http://www.linux.org'. + + If you know of additional TeX CD-ROM distributions to add to this +list, please inform <kb@mail.tug.org>. + +Tape distribution +- ----------------- + + You can obtain a complete TeX distribution, including Web2c, on tape. +Contact: + Pierre MacKay / Denny Hall, Mail Stop DH-10 / Department of Classics + University of Washington / Seattle, WA 98195 / USA + phone: 206-543-2268; email: <unixtex@u.washington.edu> + + At this writing, the distribution is available in `tar' format on 1/4 +inch 4-track QIC-24 cartridges and 4mm DAT cartridges, and the cost is +US$210. Make checks payable to the University of Washington, drawn on +a U.S. bank. Purchase orders are acceptable, but they carry an extra +charge of $10 to pay for invoice processing. Overseas sites, please +add $20 for shipment via air parcel post, or $30 for shipment via +courier. + +Other TeX packages +- ------------------ + + Many other TeX implementations are available in `CTAN:/systems', +including ready-to-run distributions for Unix, Amiga, Acorn, VMS, +Macintosh, DOS, and Windows (in various forms). Although Web2c has +support in the source code for many operating systems, and in fact some +of the other distributions are based on it, it's unlikely to work as +distributed on anything but Unix. (Please contribute improvements!) + + The Unix distribution alluded to above is the teTeX distribution. +This includes both complete sources and precompiled binaries for many +popular Unix variants, including Linux. It is based on Web2c, and +contains many other TeX-related programs as well. + + The host labrea.stanford.edu is the original source for the files for +which Donald Knuth is directly responsible: `tex.web', `plain.tex', +etc. However, unless you want to build your TeX library tree ab initio, +it is more reliable and less work to retrieve these files as part of +the above packages. In any case, labrea is not the canonical source for +anything except what was created by Stanford TeX project, so do not +rely on all the files available at that ftp site being up-to-date. + + +CTAN mirrors: + +prompt$ finger ctan@ftp.tug.org +[tug.cs.umb.edu] +... +Plan: +In order to reduce network load, it is recommended that you use the +Comprehensive TeX Archive Network (CTAN) host which is located in the +closest network proximity to your site. Alternatively, you may wish to +obtain a copy of the CTAN via CD-ROM (see help/CTAN.cdrom for details). + +Known mirrors of the CTAN reside on (alphabetically): + cis.utovrm.it (Italia) /TeX + ctan.unsw.edu.au (NSW, Australia) /tex-archive + dongpo.math.ncu.edu.tw (Taiwan) /tex-archive + ftp.center.osaka-u.ac.jp (Japan) /CTAN + ftp.ccu.edu.tw (Taiwan) /pub/tex + ftp.cdrom.com (West coast, USA) /pub/tex/ctan + ftp.comp.hkbu.edu.hk (Hong Kong) /pub/TeX/CTAN + ftp.cs.rmit.edu.au (Australia) /tex-archive + ftp.cs.ruu.nl (The Netherlands) /pub/tex-archive + ftp.cstug.cz (The Czech Republic) /pub/tex/CTAN + ftp.duke.edu (North Carolina, USA) /tex-archive + ftp.ee.up.ac.za (South Africa) /tex-archive + ftp.funet.fi (Finland) /pub/TeX/CTAN + ftp.gwdg.de (Deutschland) /pub/dante + ftp.jussieu.fr (France) /pub4/TeX/CTAN + ftp.loria.fr (France) /pub/unix/tex/ctan + ftp.mpi-sb.mpg.de (Deutschland) /pub/tex/mirror/ftp.dante.de + ftp.nada.kth.se (Sweden) /pub/tex/ctan-mirror + ftp.rediris.es (Espa\~na) /mirror/tex-archive + ftp.rge.com (New York, USA) /pub/tex + ftp.riken.go.jp (Japan) /pub/tex-archive + ftp.tu-chemnitz.de (Deutschland) /pub/tex + ftp.u-aizu.ac.jp (Japan) /pub/tex/CTAN + ftp.uni-augsburg.de (Deutschland) /tex-archive + ftp.uni-bielefeld.de (Deutschland) /pub/tex + ftp.unina.it (Italia) /pub/TeX + ftp.uni-stuttgart.de (Deutschland) /tex-archive (/pub/tex) + ftp.univie.ac.at (\"Osterreich) /packages/tex + ftp.ut.ee (Estonia) /tex-archive + ftpserver.nus.sg (Singapore) /pub/zi/TeX + kadri.ut.ee (Estonia) /pub/tex + src.doc.ic.ac.uk (England) /packages/tex/uk-tex + sunsite.cnlab-switch.ch (Switzerland) /mirror/tex + sunsite.icm.edu.pl (Poland) /pub/CTAN + sunsite.queensu.ca (Canada) /pub/tex-archive + sunsite.unc.edu (North Carolina, USA) /pub/packages/TeX + wuarchive.wustl.edu (Missouri, USA) /packages/TeX + +Known partial mirrors of the CTAN reside on (alphabetically): + ftp.adfa.oz.au (Australia) /pub/tex/ctan + ftp.fcu.edu.tw (Taiwan) /pub2/tex + ftp.germany.eu.net (Deutschland) /pub/packages/TeX + ftp.jaist.ac.jp /pub/TeX/tex-archive + ftp.uu.net (Virginia, USA) /pub/text-processing/TeX + nic.switch.ch (Switzerland) /mirror/tex + sunsite.dsi.unimi.it (Italia) /pub/TeX + sunsite.snu.ac.kr (Korea) /shortcut/CTAN + +Please send updates to this list to <ctan@urz.uni-heidelberg.de>. + +The participating hosts in the Comprehensive TeX Archive Network are: + ftp.dante.de (Deutschland) + -- anonymous ftp /tex-archive (/pub/tex /pub/archive) + -- gopher on node gopher.dante.de + -- e-mail via ftpmail@dante.de + -- World Wide Web access on www.dante.de + -- Administrator: <ftpmaint@dante.de> + + ftp.tex.ac.uk (England) + -- anonymous ftp /tex-archive (/pub/tex /pub/archive) + -- gopher on node gopher.tex.ac.uk + -- NFS mountable from nfs.tex.ac.uk:/public/ctan/tex-archive + -- World Wide Web access on www.tex.ac.uk + -- Administrator: <ctan-uk@tex.ac.uk> + + +(have a nice day) + + + +------------------------------ + + + +About TeXhax... + +Please send contributions to: TeXhax@tex.ac.uk + +Subscription and unsubscription requests: + send a one line mail message to TeXhax-Request@tex.ac.uk + containing only the line + subscribe texhax + or + unsubscribe texhax +If you have problems with un/subscribing, +please mail texhax-owner@nottingham.ac.uk + +For information on the TeX Users Group, please send a message to +TUG@TUG.org, or write TeX Users Group, 1850 Union Street, #1637 +San Francisco CA 94123 (phone: 1 415 982 8449, fax: 1 415 982 8559) + +Backnumbers of all the digests are stored in the Comprehensive TeX +Archive Network (CTAN) and can be retrieved on the Internet by +anonymous ftp. The hosts comprising CTAN include, among others, + ftp.dante.de (129.69.1.12) -- Germany + ftp.tex.ac.uk (128.232.1.87) -- UK +Please use your nearest server, to keep network load down. +The file /tex-archive/CTAN.sites on each of these hosts gives a +list of other sites which maintain full or partial mirrors of the CTAN. +Alternatively, finger ctan_us@ftp.shsu.edu for full details. + +TeXhax Digest back issues are filed below /tex-archive/digests/texhax/ +Keyword-In-Context indexes are filed in /tex-archive/digests/indexes/ + +A Hypermail version of TeXhax is also available on the World-Wide Web at URL +http://www.tex.ac.uk/tex-archive/digests/hyper/ + +\bye +End of TeXhax Digest V1997 #1 +***************************** + diff --git a/info/digests/texhax/97/texhax.02 b/info/digests/texhax/97/texhax.02 new file mode 100644 index 0000000000..64f4ec1fe5 --- /dev/null +++ b/info/digests/texhax/97/texhax.02 @@ -0,0 +1,410 @@ +(Message texhax/v97:2) +From texhax-digest-owner@nottingham.ac.uk Tue Feb 25 17:40:06 1997 +Received: from nottingham.ac.uk (jess.ccc.nottingham.ac.uk [128.243.40.193]) by granby.ccc.nottingham.ac.uk (8.6.12/8.6.12) with SMTP id RAA24973 for <cczdao@unix.ccc.nottingham.ac.uk>; Tue, 25 Feb 1997 17:40:06 GMT +Received: from majordom by nottingham.ac.uk with local (Exim 1.58 #10) + id 0vzQr9-0007jw-00; Tue, 25 Feb 1997 17:39:59 +0000 +From: owner-texhax-digest@nottingham.ac.uk +To: texhax-digest@nottingham.ac.uk +Subject: TeXhax Digest V1997 #2 +Reply-To: TeXhax@tex.ac.uk +Precedence: bulk +Message-Id: <E0vzQr9-0007jw-00@nottingham.ac.uk> +Date: Tue, 25 Feb 1997 17:39:59 +0000 + + +TeXhax Digest Tuesday, 25 February 1997 Volume 1997 : Number 002 + +(incorporating UKTeX Digest) + +Today's Topics: + + Majordomo results: TeXhax Digest V1997 #1 + Contents: TUGboat 17#4 + looking for Texcad + TeXhax Digest V96 #14 and TeXhax Digest V96 #15 is missing on CTAN + New LUG + +---------------------------------------------------------------------- + +From: majordomo@nottingham.ac.uk +Date: Sat, 8 Feb 1997 19:30:24 +0000 +Subject: Majordomo results: TeXhax Digest V1997 #1 + +- -- + +>>>> TeXhax Digest Saturday, 8 February 1997 Volume 1997 : Number 001 +**** Command 'texhax' not recognized. +>>>> +>>>> (incorporating UKTeX Digest) +**** Command '(incorporating' not recognized. +>>>> +>>>> Today's Topics: +**** Command 'today's' not recognized. +>>>> +>>>> Re> TeXhax digest +**** Command 're>' not recognized. +>>>> Testing if string starts with particular letter or word +**** Command 'testing' not recognized. +>>>> Gothic Letters in LaTeX +**** Command 'gothic' not recognized. +>>>> Where can I find the psboxit package? +**** Command 'where' not recognized. +>>>> TeX directory structure (tds) 0.9993 available +**** Command 'tex' not recognized. +>>>> web2c 7.0 and friends available +**** Command 'web2c' not recognized. +>>>> +>>>> ---------------------------------------------------------------------- +END OF COMMANDS +**** Help for majordomo: + +This is Brent Chapman's "Majordomo" mailing list manager, version 1.93. + +In the description below items contained in []'s are optional. When +providing the item, do not include the []'s around it. + +It understands the following commands: + + subscribe [<list>] [<address>] + Subscribe yourself (or <address> if specified) to the named <list>. + + unsubscribe [<list>] [<address>] + Unsubscribe yourself (or <address> if specified) from the named <list>. + + get [<list>] <filename> + Get a file related to <list>. + + index [<list>] + Return an index of files you can "get" for <list>. + + which [<address>] + Find out which lists you (or <address> if specified) are on. + + who [<list>] + Find out who is on the named <list>. + + info [<list>] + Retrieve the general introductory information for the named <list>. + + lists + Show the lists served by this Majordomo server. + + help + Retrieve this message. + + end + Stop processing commands (useful if your mailer adds a signature). + +Commands should be sent in the body of an email message to +"majordomo"or to "<list>-request". + +The <list> parameter is only optional if the message is sent to an address +of the form "<list>-request". + + +Commands in the "Subject:" line NOT processed. + +If you have any questions or problems, please contact +"majordomo-owner". + + + + +------------------------------ + +From: AMS Technical Support <tech-support@MATH.AMS.ORG> +Date: Fri, 14 Feb 1997 16:37:50 -0500 (EST) +Subject: Contents: TUGboat 17#4 + +The final issue of 1996 -- TUGboat 17#4 -- is at the printer, and +should be mailed to TUG members in a couple of weeks. In the meantime, +here is a listing of the contents, to whet your appetites. + + TUGboat + Volume 17, Number 4 / December 1996 + ==================================== + +Addresses 335 +TUG'97 Update 336 + +General Delivery + Michel Goossens 337 + From the President + Barbara Beeton + Editorial Comments 340 + A historical perspective; The Kyoto Prize for Knuth; + Recommended reading: Why things bite back; + Why does a lion represent TeX? + + Amsterdam, 13 March 1996 -- 342 + Knuth meets NTG members + CSTUG, Charles University, Prague, 355 + March 1996 --Questions and answers with + Prof. Donald E. Knuth + +Survey + Philip Taylor + Computer typesetting or electronic publishing? + New trends in scientific publication 367 + +Software & Tools + S.A. Strelkov and G.R. Epshtein + Making indices for VINITI's + ``Mathematics'' abstract journal 382 + +Philology + Mikhail Ivanovich Grinchuk + TeX and Russian traditions of typesetting 385 + +Humanities + Christina Thiele + TeX and the Humanities 388 + +Book Review + Jonathan Fine + The Printing Revolution in Early Modern + Europe, by Elizabeth L. Eisenstein 393 + +Queries + George Graetzer + Suggestions wanted for new edition of + Math into LaTeX 396 + Dirk Kuypers + PPCHTEX molecule templates 396 + +Abstracts + Les Cahiers GUTenberg, Nos. 21--24 397 + +News & Announcements + Calendar 403 + Mimi Burbank + Production notes 405 + Future issues 405 + +TUG Business + Mimi Burbank + Facts and Figures 406 + Institutional members 409 + +Forms + TUG membership application 410 + +Advertisements + TeX consulting and production services 411 + Index of advertisers 411 + + ================================================== + + + +------------------------------ + +From: "Sergey V.Tupica" <svt@yspu.yar.ru> +Date: Fri, 21 Feb 1997 12:32:35 +0300 +Subject: looking for Texcad + +Hi! + +I'm looking for latest version of TeXcad +Where can I get it? + +SvT + + + +------------------------------ + +From: volker@Illuminatus.MZ.Rhein-Main.DE (Volker Schmidt) +Date: Sun, 23 Feb 1997 10:12:24 +0100 (CET) +Subject: TeXhax Digest V96 #14 and TeXhax Digest V96 #15 is missing on CTAN + +Dear Moderator, + +I am afraid telling you for completeness that TeXhax Digest V96 +#14 and #15 is missing on the CTAN archives. + +As I am subscribed to the TeXhax mailing list I noted, that +TeXhax Digest V96 #14 is also missing from circulation. + +Could you please kindly remail me an #14 issue? Or better find +someone who can update the CTAN archive? + +TIA, +- --volker +- -- +PGP public key available on request. + +UNIX is friendly, but it chooses it's friends! + + + +------------------------------ + +From: Apostolos Syropoulos <apostolo@platon.ee.duth.gr> +Date: Tue, 25 Feb 1997 19:26:01 +0200 +Subject: New LUG + +%The English text follows the Greek text. +%%%%%%%%%%%%%%%%%%%%%%%%%%%%%%%%%%%%%%%% +% ELLHNIKA SUMFWNA ME TO BABEL +% + Agaphto'i f'iloi kai qr'hstec tou \TeX 'Ellhnec kai mh, + + Me meg'alh mac euqar'isthsh s'ac anakoin'wnoume thn 'idrush tou sull'ogou +twn Ell'hnwn F'ilwn tou \TeX ($\varepsilon\varphi\tau$). O s'ullogoc mac +aposkope'i sthn di'adosh tou \TeX gia thn stoiqeiojes'ia twn ellhnik'wn, sthn +Ell'ada kai sto exwterik'o. Ep'ishc pr'okeitai na sundr'amei sthn belt'iwsh +kai efarmog'h sta ellhnik'a dedom'ena twn ergale'iwn tou sust'umatoc \TeX kai +sthn dhmiourg'ia n'ewn ergale'iwn (p.q. tou Wm'ega) pou ja dieukol'unoun +thn paragwg'h ent'upwn sthn ellhnik'h gl'wssa (arqa'ia kai n'ea). T'eloc, +pr'ojesh mac e'inai ep'ishc h mel'eth, diat'hrhsh kai efarmog'h twn axi'wn +thc paradosiak'hc ellhnik'hc tupograf'iac sthn epoq'h twn H/U. + + Oiosd'hpote hlik'iac 18 kai 'anw, kai s'umfwnoc me to katastatik'o tou +sull'ogou, mpore'i na g'inei m'eloc tou sull'ogou. Ja pr'epei gia aut'o +na ste'ilei ston pr'oedro tou sull'ogou enup'ografh epistol'h sthn opo'ia +na dhl'wnei 'oti epijume'i na g'inei m'eloc kai 'oti ekplhre'i tic +parap'anw pro"upoj'eseic. Ep'ishc h epistol'h aut'h pr'epei na peri'eqei +to onomatep'wnumo, thn die'ujunsh, ton arijm'o thlef'wnou 'h thn +hlektronik'h ie'ujunsh tou/thc endiaferom'enou/nhc. Ta apodekt'a m'elh +plhr'wnoun t'elh eggraf'hc pou an'erqontai sto pos'o twn 5.000 draqm'wn +('h to is'oposo se 'allo n'omisma). 'Ola ta m'elh plhr'wnoun examhna'ia +sundrom'h 3.000 draqm'wn. Ta pos'a aut'a mporo'un na katablhjo'un e'ite +me taqudromik'h epitag'h 'h me metab'ibash (epikoinwn'hste me ton pr'oedro +'h ton tam'ia tou sull'ogou gia perait'erw plhrofor'iec). + + O s'ullogoc proble'pei thn 'ekdosh periodiko'u --- pr'wta se hlektronik'h +morf'h (PDF kai HTML)--- kai egqeirid'iou \LaTeX; ep'ishc probl'epei na +summet'asqei sthn sunt'hrhsh tou ellhniko'u ((babel)) kai thn dhmiourg'ia +kai di'adosh ellhnik'wn grammatoseir'wn gia to \TeX 'h to Wm'ega. + +To dioikhtik'o sumbo'ulio: + +- --- Pr'oedros: + Ap'ostoloc Sur'opouloc + apostolo@platon.ee.duth.gr +- --- Antipr'oedroc: + Gi'annhc Qaral'ampouc + yannis@pobox.com +- --- Grammat'eac: + Triant'afulloc Triantafull'idhc + (01)-4133866 +- --- Tam'iac: + Gi'wrgoc Mpakal'idhc + (0541)-73478 +- --- M'eloc: + N'ikoc Nik'opouloc + nikos@zeus.csd.auth.gr + + Die'ujunsh tou sull'ogou: + + $\varepsilon\varphi\tau$ + 28hc Oktwbr'iou 366 + 671 00 X'anjh + Ell'ac + + Thl.: (0541) 28704 + +####################################################################### + + Dear (Greek or non-Greek) Fellow TeX Users, + + It is our pleasure to be able to announce th formation of +$\varepsilon\varphi\tau$ --- The Greek TeX Friends Group. This LUG is +commited to the promotion of Greek language TeX typesetting in Greece +and abroad. Moreover, the Group is commited to the enhancement and +adaptation of the TeX family of tools to Greek and the development +of new tools (in particular, Omega) that will facilitate the creation +of documents written in the Greek language (ancient or modern). It is +also our intention to work on the study and the preservation of the +values of traditional Greek typography in the digital era. + + Anobody above 18 years of age and agreing to comply to all articles +of the bylaws of $\varepsilon\varphi\tau$, can be a member of the Group. +Any person willing to become member must hand a written application to +the president of the Group, in which it must be clearly stated that he +or she wants to become a member and that he or she fulfills the admission +requirements. Furthermore, the application must include the applicant's +name and surname, address, phone number and/or e-mail address. Admitted +applicants must pay a registration fee: GRD 5,000 (about USD 20). All +members pay a subscription fee every six months, currently fixed to +GRD 3,000 (USD 12). To obtain information on payment, please write to +the president or the treasurer of the Group. + + The Group currently has plans to publish a newsletter ---initially +only in electronic form (PDF and HTML)--- and a LaTeX manual (both in +Greek); also it plans to contribute in the maintenance of Greek babel +and the release of Greek TeX/Omega fonts. + +The Executive Board of Directors + +- --- President: + Apostolos Syropoulos, + apostolo@platon.ee.duth.gr +- --- Vice-President: + Yannis Haralambous, + yannis@pobox.com +- --- Secretary: + Triantafyllos Triantafyllidis, + +30-1-4133866 +- --- Treasurer: + Yorgos Bakalidis, + +30-541-73478 +- --- Member: + Nikos Nikopoulos, + nikos@zeus.csd.auth.gr + +Address of the Group: +$\varepsilon\varphi\tau$ +366, 28th October Str. +GR-671 00 Xanthi +GREECE + +Tel.:+30 541 28704 + + + + + + +------------------------------ + + + +About TeXhax... + +Please send contributions to: TeXhax@tex.ac.uk + +Subscription and unsubscription requests: + send a one line mail message to TeXhax-Request@tex.ac.uk + containing only the line + subscribe texhax + or + unsubscribe texhax +If you have problems with un/subscribing, +please mail texhax-owner@nottingham.ac.uk + +For information on the TeX Users Group, please send a message to +TUG@TUG.org, or write TeX Users Group, 1850 Union Street, #1637 +San Francisco CA 94123 (phone: 1 415 982 8449, fax: 1 415 982 8559) + +Backnumbers of all the digests are stored in the Comprehensive TeX +Archive Network (CTAN) and can be retrieved on the Internet by +anonymous ftp. The hosts comprising CTAN include, among others, + ftp.dante.de (129.69.1.12) -- Germany + ftp.tex.ac.uk (128.232.1.87) -- UK +Please use your nearest server, to keep network load down. +The file /tex-archive/CTAN.sites on each of these hosts gives a +list of other sites which maintain full or partial mirrors of the CTAN. +Alternatively, finger ctan_us@ftp.shsu.edu for full details. + +TeXhax Digest back issues are filed below /tex-archive/digests/texhax/ +Keyword-In-Context indexes are filed in /tex-archive/digests/indexes/ + +A Hypermail version of TeXhax is also available on the World-Wide Web at URL +http://www.tex.ac.uk/tex-archive/digests/hyper/ + +\bye +End of TeXhax Digest V1997 #2 +***************************** + diff --git a/info/digests/texhax/97/texhax.03 b/info/digests/texhax/97/texhax.03 new file mode 100644 index 0000000000..98fd4f0119 --- /dev/null +++ b/info/digests/texhax/97/texhax.03 @@ -0,0 +1,215 @@ +Article 70 of ucam.mlist.texhax: +From: owner-texhax-digest@nottingham.ac.UK +Subject: TeXhax Digest V1997 #3 +Date: 21 Mar 97 10:00:50 GMT + +TeXhax Digest Friday, 21 March 1997 Volume 1997 : Number 003 + +(incorporating UKTeX Digest) + +Today's Topics: + + Re: TeXhax Digest V96 #14 and TeXhax Digest V96 #15 is missing on CTAN + Re: "LaTeX Error: no line to end here" message at first \subsection in book + Computer Modern PostScript Fonts + +---------------------------------------------------------------------- + +From: David.Osborne@nottingham.ac.uk +Date: Mon, 03 Mar 1997 10:23:14 +0000 +Subject: Re: TeXhax Digest V96 #14 and TeXhax Digest V96 #15 is missing on CTAN + +Volker Schmidt writes [23 Feb 1997 10:12:24 +0100]: + + > I am afraid telling you for completeness that TeXhax Digest V96 + > #14 and #15 is missing on the CTAN archives. + +I have now installed the missing issues in the archive. For future reference, +please note that the Majordomo list manager also archives copies of the digest, +which you could retrieve by mail: + + To: majordomo@nottingham.ac.uk + + index texhax + +to get a list of filenames, then + + To: majordomo@nottingham.ac.uk + + get texhax v96.n014 + get texhax v96.n015 + +to retrieve V96 #14 and #15 + +- -- +David Osborne email: David.Osborne@nottingham.ac.uk +(moderator, TeXhax Digest) tel: +44 (0)115 951 3397 +Cripps Computing Centre +University of Nottingham, UK + +------------------------------ + +From: "Leif Peterson, Ph.D." <peterson@bcm.tmc.edu> +Date: Wed, 05 Mar 1997 15:55:37 -0500 +Subject: Re: "LaTeX Error: no line to end here" message at first \subsection in book + +Dear texhax: + +I am using: + +tex386 ver 3.14159 [4b] +LaTeX2e <1996/06/01> +Class Book 1996/05/26 v 1.3r + +Recently, I started getting the error message "No line to end here" at the +first occurrence of a \subsection command in a book a am compiling. The +first occurrent of a \subsection is in the third chapter and when I isolate +and compile the chapter by itself, I still receive the error message. Thus, +there is obviously something going on at first recognition of a \subsection +command. I looked in the chapter for a \\ or a \newline (which I never +use), which is recommended for debugging this error, and did not find +either. I must say, that I have modified the *style* parameter in +@subsection of book.cls with a \centering command, removed it, and this did +not prevent the error message from occurring. + +Previously, I could make a full compile of the book without this message +>occurring. Is there something that I could have done to cause this error +message? How can I prevent it from occurring. + +Please help. + +Thanks, + +Leif Peterson + +------------------------------ + +From: Ralph Youngen <rey@ams.org> +Date: Fri, 14 Mar 1997 15:49:50 -0500 (EST) +Subject: Computer Modern PostScript Fonts + +The American Mathematical Society is pleased to announce the public release +of the Computer Modern PostScript Fonts in Adobe Type 1 format. The +READ.ME file that accompanies this release appears below. We call your +attention to the second full paragraph of the READ.ME file which discusses +the AMS copyright associated with this release. We hope that this +explanation will set aside any possible confusion regarding the intent of +the AMS copyright with respect to these fonts. + +Ralph Youngen +Director, Electronic Product Development +American Mathematical Society + + -------------------- + +Computer Modern PostScript Fonts +(Adobe Type 1 format) + +- ----------------------------------------------------------------------------- +The PostScript Type 1 implementation of the Computer Modern fonts produced by +and previously distributed by Blue Sky Research and Y&Y, Inc. are now freely +available for general use. This has been accomplished through the cooperation +of a consortium of scientific publishers with Blue Sky Research and Y&Y. +Members of this consortium include: + + Elsevier Science + IBM Corporation + Society for Industrial and Applied Mathematics (SIAM) + Springer-Verlag + American Mathematical Society (AMS) + +In order to assure the authenticity of these fonts, copyright will be held +by the American Mathematical Society. This is not meant to restrict in any +way the legitimate use of the fonts, such as (but not limited to) electronic +distribution of documents containing these fonts, inclusion of these fonts +into other public domain or commercial font collections or computer +applications, use of the outline data to create derivative fonts and/or +faces, etc. However, the AMS does require that the AMS copyright notice be +removed from any derivative versions of the fonts which have been altered in +any way. In addition, to ensure the fidelity of TeX documents using Computer +Modern fonts, Professor Donald Knuth, creator of the Computer Modern faces, +has requested that any alterations which yield different font metrics be +given a different name. + +The AMS does not provide technical support or installation assistance +beyond any installation instructions included in this file. Installation +and use of these fonts may require some technical expertise. Review this +READ.ME file in its entirety before undertaking an installation. + +- ---------------------------------------------------------------------------- +History + +The PostScript versions of the Computer Modern fonts were produced in 1988 +by Blue Sky Research of Portland, Oregon, and Y&Y, Inc., of Concord, +Massachusetts, who published the fonts in conjunction with their commercial +implementations of the TeX program. + +Character outlines were derived from high-resolution METAFONT-generated +character bitmaps by the ScanLab application from Projective Solutions (Ian +Morrison and Henry Pinkham), applied and corrected by Douglas Henderson of +Blue Sky Research. Character hints were created by software from Y&Y +(Berthold and Blenda Horn), with extensive hand work by Blenda Horn. Font +engineering, production, and packaging were by Douglas Henderson and +Berthold Horn. The CMMI* fonts were revised in 1996 to conform to Knuth's +changes to the greek delta and arrow characters. + +- ---------------------------------------------------------------------------- +Font Distributions + +The canonical version of the Computer Modern PostScript Fonts is located on +the AMS FTP server, e-math.ams.org, at /pub/tex/cmfonts/ps. This area is +also mirrored on the Comprehensive TeX Archive Network (CTAN) at +fonts/cm/ps-type1/bluesky. + +The following three files are in this directory for you to download: + +cmps-macintosh.hqx for use on a Macintosh, contains fonts in standard + Macintosh Type 1 format +cmps-pc.zip for use on a Windows or DOS system, contains fonts + in PFB format with PFM metrics files +cmps-unix.tar.gz for use on a Unix system, contains fonts in + PFB format with AFM metrics files + +Each distribution includes a READ.ME file which contains instructions +for installing the fonts. Please review the READ.ME file in its entirety +before undertaking to install the fonts on your system. + +------------------------------ + +About TeXhax... + +Please send contributions to: TeXhax@tex.ac.uk + +Subscription and unsubscription requests: + send a one line mail message to TeXhax-Request@tex.ac.uk + containing only the line + subscribe texhax + or + unsubscribe texhax +If you have problems with un/subscribing, +please mail texhax-owner@nottingham.ac.uk + +For information on the TeX Users Group, please send a message to +TUG@TUG.org, or write TeX Users Group, 1850 Union Street, #1637 +San Francisco CA 94123 (phone: 1 415 982 8449, fax: 1 415 982 8559) + +Backnumbers of all the digests are stored in the Comprehensive TeX +Archive Network (CTAN) and can be retrieved on the Internet by +anonymous ftp. The hosts comprising CTAN include, among others, + ftp.dante.de (129.69.1.12) -- Germany + ftp.tex.ac.uk (128.232.1.87) -- UK +Please use your nearest server, to keep network load down. +The file /tex-archive/CTAN.sites on each of these hosts gives a +list of other sites which maintain full or partial mirrors of the CTAN. +Alternatively, finger ctan_us@ftp.shsu.edu for full details. + +TeXhax Digest back issues are filed below /tex-archive/digests/texhax/ +Keyword-In-Context indexes are filed in /tex-archive/digests/indexes/ + +A Hypermail version of TeXhax is also available on the World-Wide Web at URL +http://www.tex.ac.uk/tex-archive/digests/hyper/ + +\bye + +End of TeXhax Digest V1997 #3 +***************************** diff --git a/info/digests/texhax/97/texhax.04 b/info/digests/texhax/97/texhax.04 new file mode 100644 index 0000000000..89414f4cef --- /dev/null +++ b/info/digests/texhax/97/texhax.04 @@ -0,0 +1,337 @@ +Article 71 of ucam.mlist.texhax: +From: owner-texhax-digest@nottingham.ac.UK +Subject: TeXhax Digest V1997 #4 +Date: 14 Apr 97 10:48:50 GMT + +TeXhax Digest Monday, 14 April 1997 Volume 1997 : Number 004 + +(incorporating UKTeX Digest) + +Today's Topics: + + Directory searching by TeX + Two Columns for two "documents" + [none] + Writing to Files in Latex2e + How to remove bitmap-ed cm fonts? + notice regarding tex users group election + texhax + +---------------------------------------------------------------------- + +From: hkim@pav.research.panasonic.com (Hee-Yong Kim) +Date: Fri, 21 Mar 1997 11:28:02 -0500 +Subject: Directory searching by TeX + +Hellow Tex Hackers, + +Is there any way to tell "Tex" to search a current directory. +Thing is that when I run our "Tex" program (through Latex), +it did not search files in the current directory. + -------- -------- +So, I have to tell Tex all the complete path for the input files, +and there is a serious problem when "Tex" look for "AUX" file for the cross +reference, +because it could not find it even if the file was in the current directory +after the previous run. + +It might be a flaw of our"Tex" program. +Perhaps, when it was created, search paths were passed on it, +but the current directory was omitted. +Is there any quick fix, or I have to install brand new Tex program ? + +Thank you. + +hkim@pav.research.pansonic.com + +------------------------------ + +From: D Cook <D.Cook@sheffield.ac.uk> +Date: Mon, 24 Mar 1997 09:16:52 +0000 (GMT) +Subject: Two Columns for two "documents" + +Dear TeXhax, +This may be common knowledge but not, I'm afraid, to me. +Is there a cls file or a package which enables one to generate a document +in which the pages are two columns over the whole document but the +contents of the two columns are basically two different source files? +I am thinking of, for example, a document and its translation to another +language or a document and a running commentry printed alongside and "in +step" with each other. The existing multicolumn methods are basically for +re-formatting a single continuous whole and I guess could be made to do +what I want but it would be tremendously inconvenient keeping things in +step. +Thanks in advance +Dave Cook + +Dr. David B. Cook +The Dept. of Chemistry +The University of Sheffield +SHEFFIELD +S3 7HF UK +phone +44 (0)114 2824448 +FAX +44 (0)114 2738673 +http://spider.shef.ac.uk/ + +========================================================================= +Remember, + +Establishment fame is poor compensation for a lifetime of boredom, + + Do your own thing; + your reward will be doing it, + your punishment having done it + +========================================================================= + +------------------------------ + +From: "Tracy, Michael J" <Michael.Tracy@PSS.boeing.com> +Date: Tue, 1 Apr 1997 07:54:26 -0800 +Subject: [none] + +hello, +I am hoping that you might be able to help me with a TEX question. I am +sure this question has been asked before, but do you know how to create +bold lower case Greek letters in math mode? I have done it before in +LATEX using \boldmath and \unboldmath but these commands are not +available in TEX. + +thank you for your help + +mike tracy +Boeing Commercial Space +Seattle WA + +------------------------------ + +From: reynoldd@ccmail.dcu.ie +Date: Mon, 07 Apr 1997 15:42:27 +0000 (GMT) +Subject: Writing to Files in Latex2e + + I would like to be able to write to a file within LateX2e, and can see + nothing about this in the LaTeX Companion. I would prefer not to have + to revert to the plain TeX commands \write and \shipout unless + necessary. + + In particular, I'm writing macros for a problem sheet, and would like + to use a format like + + \begin{question} + What is the capital of UK? + \end{question} + \begin{solution} + London + \end{solution} + + \begin{question} + What is the capital of Republic of Ireland? + \end{question} + \begin{solution} + Dublin + \end{solution} + + etc. + + Certain counters are incremented by the question environment, and I'd + like to format the solution using these values before moving on the + next question, but not print it there. One way to do this would be to + get the solution environment within test.tex to write to another file + solution.tex or test.sol, and run LaTeX on this automatically produced + file; or just to have the solutions written to the auxiliary file, + like references, and cite them all at the end of document. Any + suggestions on what would be a good solution and how to implement it + would be much appreciated. + + David Reynolds + +------------------------------ + +From: "Yaroslav S. Ussenko" <ysu@iist.unu.edu> +Date: Tue, 08 Apr 1997 14:47:00 +0800 +Subject: How to remove bitmap-ed cm fonts? + +Dear all, + +Is there any program which can remove bitmap-ed cm fonts +from PostScript files generated by dvips and add references to +type-1 outlines of cm fonts instead? + +Thanks in advance, +Yarick. + +------------------------------ + +From: bbeeton <BNB@MATH.AMS.ORG> +Date: Wed, 09 Apr 1997 09:31:08 -0400 (EDT) +Subject: notice regarding tex users group election + + This notice was sent by email on 5 April 1997 to all TUG members + for whom TUG has email addresses. If anyone reading this notice + here is a member, but did not already receive a copy by email, + please let us know by sending a message to tug@mail.tug.org + including your correct, preferred email address. We intend to + use email more frequently in the future to communicate with + TUG members, and need your help to keep the records up to date. + + -------------------- + + Notice regarding 1997 TeX Users Group election + +The 1997 election for the TeX Users Group Board of Directors has +now ended. Since there were fewer nominated candidates than vacant +positions, in accordance with the Election Procedures, the candidates +who did submit nomination papers are declared elected. No ballots +will be circulated. + +The open positions, as announced, were President and six members +of the Board of Directors. + +The following individuals will be taking positions on the Board +of Directors for 4-year terms ending in 2001: + + Donna Burnette + Mimi Jett + Patricia Monohon + Arthur Ogawa + Petr Sojka + +In the absence of a ballot, the statements of candidacy which would +have appeared there will be published instead in TUGboat 18#1. + +Four Board members whose terms were due to end in 1998 had agreed +that they would be willing for their terms to be extended to 1999 +to accommodate a change in the election schedule from annually to +every two years (see the election announcement, TUGboat 17#3, p.240). +Had there been a ballot, their names would have been listed there +for confirmation by the membership. However, in the absence of a +ballot, it is proposed that, in accordance with the Bylaws (Article +VII, Section 10), the incoming President confirm the extension +of their terms. These Board members are: + + Barbara Beeton + Karl Berry + Judy Johnson + Jiri Zlatuska + +There is no candidate for TUG President. This poses a problem. +Although the Bylaws and Election Procedures provide for the filling +of vacant Board positions by appointment, no provision exists for +the absence of a candidate for President. There is not sufficient +time left before the annual meeting in July to solicit additional +candidates and circulate ballots. + +In order to ensure that the necessary decisions are arrived at in +a democratic manner, and that the incoming Board has some say in +the matters which will affect them most directly, Michel Goossens, +the outgoing President, has appointed the new members to the Board +for a temporary period effective immediately, to expire when their +regular term begins. + +It is proposed that individuals wishing to present themselves as +candidates for President do so in the following manner, with the +election be held during the annual TUG business meeting which will +take place during the TUG annual meeting in San Francisco +(July 28 - August 1). + +Any TUG member in good standing who will be present at the annual +business meeting may submit a valid nomination form and supporting +statement (see the announcement in TUGboat 17#3 or the TUG Web pages, +http://www.tug.org) no later than Monday, July 28 (the first day +of the meeting), and be prepared to present his/her program at the +business meeting, (currently scheduled for Thursday, July 31). + +(All TUG members are reminded that except for certain transactions +such as personnel matters, which legally are privileged, Board +meetings are open to members as observers.) The Board meeting is +scheduled to be held on Saturday and Sunday, July 26-27, at the main +meeting site; room details will be posted when available. Candidates +for President are encouraged to attend. + +Nomination forms and supporting documentation should be submitted to +the Elections Committee by one of two methods: (1) by July 15, sent +to the address below rather than to the address given in the TUGboat +announcement, or (2) July 26-28, brought to the meeting and delivered +in person to one of the undersigned committee members. If sending +forms by mail or fax, please confirm this to the Committee at the +email address below; electronic copies of candidates' statements +may also be sent to this address any time before the meeting. + +In order for this business to be transacted legally, at least 50 TUG +members must be present to form a quorum (Bylaws, Article III, Section 6). +It is therefore very important for members to attend this meeting, in order +to help define the future of our organization. + + For the Elections Committee + Sebastian Rahtz, Barbara Beeton + +Address for submission of nominations for TUG President: +Barbara Beeton +TUG Elections Committee +American Mathematical Society +P.O. Box 6248 +Providence, RI 02940 +Fax: +1 401 331-3842 +Email: tug-election@mail.tug.org + +------------------------------ + +From: Mona.Jacobsen@termo.unit.no +Date: Mon, 14 Apr 97 08:54:27 +0200 +Subject: texhax + +BibTeX +Can somebody help me with a style which makes it possible to have a +bibliography +at the end of each chapter in a book. As far as I have experienced, LaTex +allows only one bibliography in the document........ + +LaTeX version: +This is TeX, C Version 3.141 +(artfl_97.tex +LaTeX Version 2.09 <25 March 1992> + +BibTeX version: +This is BibTeX, C Version 0.99c + +Mona Jacobsen + +------------------------------ + +About TeXhax... + +Please send contributions to: TeXhax@tex.ac.uk + +Subscription and unsubscription requests: + send a one line mail message to TeXhax-Request@tex.ac.uk + containing only the line + subscribe texhax + or + unsubscribe texhax +If you have problems with un/subscribing, +please mail texhax-owner@nottingham.ac.uk + +For information on the TeX Users Group, please send a message to +TUG@TUG.org, or write TeX Users Group, 1850 Union Street, #1637 +San Francisco CA 94123 (phone: 1 415 982 8449, fax: 1 415 982 8559) + +Backnumbers of all the digests are stored in the Comprehensive TeX +Archive Network (CTAN) and can be retrieved on the Internet by +anonymous ftp. The hosts comprising CTAN include, among others, + ftp.dante.de (129.69.1.12) -- Germany + ftp.tex.ac.uk (128.232.1.87) -- UK +Please use your nearest server, to keep network load down. +The file /tex-archive/CTAN.sites on each of these hosts gives a +list of other sites which maintain full or partial mirrors of the CTAN. +Alternatively, finger ctan_us@ftp.shsu.edu for full details. + +TeXhax Digest back issues are filed below /tex-archive/digests/texhax/ +Keyword-In-Context indexes are filed in /tex-archive/digests/indexes/ + +A Hypermail version of TeXhax is also available on the World-Wide Web at URL +http://www.tex.ac.uk/tex-archive/digests/hyper/ + +\bye + +End of TeXhax Digest V1997 #4 +***************************** diff --git a/info/digests/texhax/97/texhax.05 b/info/digests/texhax/97/texhax.05 new file mode 100644 index 0000000000..04802d6719 --- /dev/null +++ b/info/digests/texhax/97/texhax.05 @@ -0,0 +1,434 @@ +Article 72 of ucam.mlist.texhax: +From: owner-texhax-digest@nottingham.ac.UK +Subject: TeXhax Digest V1997 #5 +Date: 27 May 97 12:00:06 GMT + +TeXhax Digest Tuesday, 27 May 1997 Volume 1997 : Number 005 + +(incorporating UKTeX Digest) + +Today's Topics: + + multi-page tables/arrays + passing a parameter to latex + Numbering figures in LaTeX + Re: Two Columns for two "documents" + Hanging punctuation + Re: Spell Checker for Windows Users who TeX + +---------------------------------------------------------------------- + +From: Robert Bruner <rrb@math.wayne.edu> +Date: Mon, 14 Apr 97 12:48:48 EDT +Subject: multi-page tables/arrays + +Dear TeXperts, + +I have 140 tables/arrays (I am willing to use either), of varying +sizes, from 1/6 of a page to 4 or 5 pages in length, in a document I am +producing. I would like them to appear one after another with only a +small break between them. The behavior of the array and table +environments makes this difficult, however. + +I need an array-like environment which would split the array across +pages as necessary, and would preferably add a note saying "continued" +at the bottom and a header labelling the array at the top of each +continuation page. + +The array environment (and table also) insists on putting the entire +array on one page. I have manually inserted the end array- begin array +commands to break it, but this is tedious and doesn't produce optimal +results. + +I could use tabbing, but would not get the headers or the "continued" +notes (except with manual insertion of them). + +I am using AMSLaTeX, and would be willing to switch to +LaTeX or LaTeX2e to solve this. Learning plain TeX or AMSTex to +do this would take longer, but if that's what it takes, so be it. + +Robert Bruner +Department of Mathematics +Wayne State University +Detroit, Michigan USA 48202 +rrb@math.wayne.edu + +------------------------------ + +From: "Cliff Bergman" <cbergman@iastate.edu> +Date: Mon, 14 Apr 1997 12:42:37 CDT +Subject: passing a parameter to latex + +Hello, + +I have recently converted from amstex to latex2e. I work on a UNIX +system. I have a question. I would like to selectively load a package +depending on a parameter that I pass on the command line. This seems to +work: + +\documentclass{article} +\iffoo \usepackage{xyzpackage} \fi +\begin{document} + +and then the command line: + latex '\newif\iffoo \foofalse \input testfile' +will run the file and will cause the package not to be loaded. + +This is only slightly unaesthetic, since I have to define the switch as +well as set it on the command line. A better solution would be to +create my own format file containing the switch. However I have been +unsuccessful at creating a format built on top of latex2e. (I tried the +'mylatex' method, but it did not work.) + +Does anyone know how create such a format file, or does anyone know of +an obscure switch sitting around in latex that I could 'borrow' (at my +own risk!) for this purpose? Or is there another way to implement my +scheme? + +Thanks in advance, +cliff bergman + +cbergman@iastate.edu + +------------------------------ + +From: Mona.Jacobsen@termo.unit.no +Date: Thu, 17 Apr 97 13:17:54 +0200 +Subject: Numbering figures in LaTeX + +Is there a possibility to number figures and tables in report style with one +level (Figure 1, Figure 2 etc...) and not (Figure 1.1 Figure 1.2 etc...) + +LaTeX version: +This is TeX, C Version 3.141 +(artfl_97.tex +LaTeX Version 2.09 <25 March 1992> + +BibTeX version: +This is BibTeX, C Version 0.99c + +Mona Jacobsen + +------------------------------ + +From: Jonathan Fine <J.Fine@pmms.cam.ac.uk> +Date: Fri, 18 Apr 97 18:13 BST +Subject: Re: Two Columns for two "documents" + +This is a response to a query in TeXHax. 18 April 1997 +Dear TeXhax, +David Cook (D.Cook@sheffield.ac.uk) asks: +- ---------------------------------------------------------------------- +Is there a cls file or a package which enables one to generate a document +in which the pages are two columns over the whole document but the +contents of the two columns are basically two different source files? +- ---------------------------------------------------------------------- + +Briefly, the answer to this question is no, and for reasons connected +to the architecture of TeX (the program) and LaTeX. It is not +possible, with \input, to read a bit from one file and then a bit +from another, alternately. This is the way things are. + +It is possible, with \read, to alternate between two (or more) files, +but this would have the side-effect of rendering dysfunctional all +macros that depend on changing of \catcode's. + +My advice is to use some external program or utility to weave +together the two different source files, and then use this new file +as the input to TeX (or LaTeX). + +He also writes: +- ---------------------------------------------------------------------- +The existing multicolumn methods are basically for re-formatting a +single continuous whole and I guess could be made to do what I want +but it would be tremendously inconvenient keeping things in step. +- ---------------------------------------------------------------------- + +Suppose that the \input problem has been solved. Forming the two +files into two separate galleys should not be very difficult, but +deciding on page breaks will be hard. Here is one way to do this. +First set each file by itself, at the column measure. Print out the +results, and compare. On the basis of this, decide where the page +breaks should be. Now use \vadjust or the like to force these page +breaks (say in conjunction with \vsplit) in the source file, and use +a custom output routine to assemble the pages up. Whatever algorithm +one uses, one will probably have to resort to such hand tuning to get +a decent result. So let that be the first `algorithm'. + +I'm sorry that there is probably not a LaTeX package for doing this, +so far as I know. It would probably take a wizard to write one. + +sincerely + +Jonathan Fine +Mailing Address: 203 Coldhams Lane, Cambridge, CB1 3HY +Telephone: 01223 215389 + +------------------------------ + +From: Marcus Vinicius Mesquita de Sousa <sousa@ifi.unicamp.br> +Date: Thu, 24 Apr 1997 20:29:56 -0300 +Subject: Hanging punctuation + +For TeXperts only! + +In appendix D from the TeXbook, p. 394, DEK +gives some macros in order to get TeX typesetting +with hanging punctuation. With some minor modifications +and inclusions (given in the end of this mail), +these macros work fine with LaTeX2e, +except for the hyphen! According to DEK, you shoud +have a special font with a zero width \hyphenchar +and that is not the case with the font I use (Stempel Garamond). + +Well, with the macro \hyphdisc + +\newdimen\hyphenhang +\setbox0=\hbox{-} +\hyphenhang=\wd0 + +\def\hyphdisc{\ifhmode\allowhyphens% +\discretionary{-\kern-2\hyphenhang\kern\hyphenhang}% +{}{}\allowhyphens\fi} + +I can get hanging hyphens, but I must introduce them +where they appear after a first LaTeX running +via the macro \hyphdisc . + +So, for example + +blablabla\hyphdisc blablabla blablabla + +gives + +blablabla- +blablabla +blablabla + +My question is: is there any way to assign the macro +\hyphdisc to the \defaulthyphenchar ?? + +Marcus Sousa +State University of Campinas +E-mail: sousa@ifi.unicamp.br + +********************************************** +% File hang.sty +% File for ``hangin punctuation'' for LaTeX +% Most macros taken from TheTeXbook, p. 395 +% with some minor modifications and inclusions to work with LaTeX + +\makeatletter +\def\allowhyphens{\penalty\@M\hskip\z@skip} +\makeatother + +% Period +\newdimen\periodhang +\setbox0=\hbox{.} +\periodhang=\wd0 +\def\period{\ifhmode.\kern-\periodhang\kern\periodhang% +\else\string.\fi} +\catcode`.=\active +\let.=\period + +% comma +\newdimen\commahang +\setbox0=\hbox{,} +\commahang=\wd0 +\def\comma{\ifhmode,\kern-\commahang\kern\commahang% +\else\string,\fi} +\catcode`,=\active +\let,=\comma +\commahang=0.2em +\def\commakern{\kern-\commahang} + +% colon +\newdimen\colonhang +\setbox0=\hbox{:} +\colonhang=\wd0 +\def\colon{\ifhmode:\kern-\colonhang\kern\colonhang% +\else\string:\fi} +\catcode`:=\active +\let:=\colon + +% semicolon +\newdimen\semicolonhang +\setbox0=\hbox{;} +\semicolonhang=\wd0 +\def\semicolon{\ifhmode;\kern-\semicolonhang\kern\semicolonhang% +\else\string;\fi} +\catcode`;=\active +\let;=\semicolon + +% Interrogation mark +\newdimen\imarkhang +\setbox0=\hbox{?} +\imarkhang=\wd0 +\def\imark{\ifhmode?\kern-\imarkhang\kern\imarkhang% +\else\string?\fi} +\catcode`?=\active +\let?=\imark + +% Exclamation mark +\newdimen\emarkhang +\setbox0=\hbox{!} +\emarkhang=\wd0 +\def\emarkkern{\kern-\emarkhang} +\def\emark{\ifhmode!\kern-\emarkhang\kern\emarkhang% +\else\string!\fi} +\catcode`!=\active +\let!=\emark + +% Definition of \hyphdisc as a control word: + +\newdimen\hyphenhang +\setbox0=\hbox{-} +\hyphenhang=\wd0 + +\def\hyphdisc{\ifhmode\allowhyphens\discretionary{-\kern-2\hyphenhang\kern\hyphenhang}{}{}\allowhyphens\fi} + +% Special treatment for quotes + +\newdimen\quotehang +\setbox0=\hbox{`} +\quotehang=\wd0 +\newdimen\qquotehang +\setbox0=\hbox{``} +\qquotehang=\wd0 + +\def\lqq{``} +\def\rqq{''} + +\def\rquote{'\kern-\quotehang\kern\quotehang} +\def\lquote{\ifhmode\kern\quotehang\vadjust{}\else\leavevmode\fi% + \kern-\quotehang`\allowhyphens} + +\catcode`'=\active +\def'{\futurelet\next\rqtest} +\catcode``=\active +\def`{\futurelet\next\lqtest} + +\def\rqtest{\ifx\next'\let\next=\rquotes\else\let\next=\rquote\fi\next} +\def\lqtest{\ifx\next`\let\next=\lquotes\else\let\next=\lquote\fi\next} + +\def\rquotes'{\rqq\kern-\qquotehang\kern\qquotehang} +\def\lquotes`{\ifhmode\kern\qquotehang\vadjust{}% + \else\leavevmode\fi% + \kern-\qquotehang\lqq\allowhyphens} + +------------------------------ + +From: Jaime Cuevas Dermody <dermody@xjcd.prestel.co.uk> +Date: Thu, 01 May 1997 04:06:47 +0100 +Subject: Re: Spell Checker for Windows Users who TeX + +Hello, + +I am an experienced TeXer who uses plain TeX via the commercial YandY TeX +package. There must be a lot of TeX users who face the same problems as I +and whose productivity would be enhansed if they could be solved +efficiently. + +I was browsing through your Web Site and got to a page on spell checkers, +where I found recommendations on spell checkers for people using UNIX, +Macintoish,DOS, and even VMS. But I, like 90% of the PC users and most +TeXers, use Windows 95. What recommendations do you have for spell +checkers that work well in a Windows 95 enviornment. + +While we are on the topic of programs to use with TeX, what is there for +footnotes. I now use a combersome big macro called Eplain, that has it +origins in Gnu. I only use the footnote part of that macro. It gives +me easy control over footnotes, e.g., I can control the vertical space +between +sucessive footnotes on the same page. But it always puts a little extra +baselineskip just above the last line of each footnote. Six years ago, I +spent many hours trying to write my own footnote little macro, by modifying +the original TeX instruction for \footnote, but could never get it to run. + +I use Microsoft Word and its spell checker to prepare ASCII TeX files, but +have found no way to get it to avoid leading \ characters. I +would actually prefer for a spell checker to recognize and remember lead \ +characters so I could detect errors in TeX commands, via-a-via an +accumulated dictionary of TeX commands. But Word and most editos do not +store \. + +I would gladly switch to a good quality simple ASCII text editor or another +word processor, if I could find one that worked well with a good spell +checker. I tried Epsilon and found it to be very cumbersome and +inappropriate. It is optimal only for programmers with near repititions of +large code sections. It requires far to many key strokes and attention per +page to type a TeX file. I would like to be able to control the size and +style of font in the edit window of the text editor, so I can take +advantage of my 1600X1200-pixel 21-inch monitor. Word does at least that. + +I use PicTeX alot and am very frustrated that I cannot rotate text so that +it can follow a slanted line or a curve. I may have to draw my diagrams in +Adobe Illustrator and then use encapulated postscript to insert TeX text +along lines or curves. Unfortunately that is a lot of work and requires me +to become adept a Adobe Illustrator, which is like getting married. Is +there a TeX Cad program that will allow me to draw functions and place +labels along the functions. About five years ago, I tried a TeX CAD macro +and found it far too primitive. As I recall it came with EM TeX.tug. + +I downloaded pSTricks and found I could not TeX any of its files. It has @ +characters where I am used to seeing \ charaters. The signal not to try +to use it came form the note that only the part of the manual pertaining to +the latest change was included, and that one need ot get all the manuals of +previuous version to have a complete manual. That is a scarry to invite +thousands of to search for the same set of manuals instead of one person, who +knows far better where they all are, manking them available for downloading. +I had a nice talk with the author of TeXDraw and found that macro to be a +great package oriented to Unix rather than Windows 95. + + Cheers, + Jaime + +- ---------------------------------------------------------------------- +Dr. Jaime Cuevas Dermody, Senior Lecturer +University of Strathclyde Department of Accounting and Finance +100 Cathedral St., Glasgow G4 0LN, Scotland PHONE: 44(0)141 548-3891 + FAX: 552-3547 +HOME: 159 Maxwell Drive PHONE: 44(0)141 419-0300 + Glasgow G41 5AE, Scotland FAX: 419-0301 +Do not dial the (0)s from outside the U.K. MOBILE: (0)385 901-226 +E-Mails: j.c.dermody@strath.ac.uk dermody@xjcd.prestel.co.uk + +------------------------------ + +About TeXhax... + +Please send contributions to: TeXhax@tex.ac.uk + +Subscription and unsubscription requests: + send a one line mail message to TeXhax-Request@tex.ac.uk + containing only the line + subscribe texhax + or + unsubscribe texhax +If you have problems with un/subscribing, +please mail texhax-owner@nottingham.ac.uk + +For information on the TeX Users Group, please send a message to +TUG@TUG.org, or write TeX Users Group, 1850 Union Street, #1637 +San Francisco CA 94123 (phone: 1 415 982 8449, fax: 1 415 982 8559) + +Backnumbers of all the digests are stored in the Comprehensive TeX +Archive Network (CTAN) and can be retrieved on the Internet by +anonymous ftp. The hosts comprising CTAN include, among others, + ftp.dante.de (129.69.1.12) -- Germany + ftp.tex.ac.uk (128.232.1.87) -- UK +Please use your nearest server, to keep network load down. +The file /tex-archive/CTAN.sites on each of these hosts gives a +list of other sites which maintain full or partial mirrors of the CTAN. +Alternatively, finger ctan_us@ftp.shsu.edu for full details. + +TeXhax Digest back issues are filed below /tex-archive/digests/texhax/ +Keyword-In-Context indexes are filed in /tex-archive/digests/indexes/ + +A Hypermail version of TeXhax is also available on the World-Wide Web at URL +http://www.tex.ac.uk/tex-archive/digests/hyper/ + +\bye + +End of TeXhax Digest V1997 #5 +***************************** diff --git a/info/digests/texhax/97/texhax.06 b/info/digests/texhax/97/texhax.06 new file mode 100644 index 0000000000..99800b4435 --- /dev/null +++ b/info/digests/texhax/97/texhax.06 @@ -0,0 +1,442 @@ +Article 99958 of comp.text.tex: +From: owner-texhax-digest@nottingham.ac.UK +Subject: TeXhax Digest V1997 #6 +Date: 25 Jul 1997 16:08:56 +0100 +Organization: None + +TeXhax Digest Friday, 25 July 1997 Volume 1997 : Number 006 + +(incorporating UKTeX Digest) + +Today's Topics: + + PCTEX + [none] + Advanced mathematical typesetting in LaTeX + TUGboat Volume 18 Issue 1 + SliTeX + cyrtug'97 + TeX directory structure (tds) 0.9993 available + TeX Graphic Package? + boxed, multiline formula + +---------------------------------------------------------------------- + +From: Edson Luiz Cataldo Ferreira <chanteur@ax.apc.org> +Date: Thu, 29 May 1997 11:35:40 -0300 (EST) +Subject: PCTEX + +>Hi, Sir ! +> +> I've just got the version LATEX 2e and i'd like to obtain informations of +>it. More precisely i'd like to obtain informations about PCTEX (version 2e). +>I'd like to know if there is publications about it. That is, books or or +>other publication that explain how to use it. I've user a version older of +>PCTEX. +> +>Thank you for you attention, +> +>Edson. + +------------------------------ + +From: sla019@pop-ei.ku-eichstaett.de +Date: Thu, 29 May 1997 21:57:44 +0000 +Subject: [none] + +> I would gladly switch to a good quality simple ASCII text editor or +> another word processor, if I could find one that worked well with a good +> spell checker. I tried Epsilon and found it to be very cumbersome and +> inappropriate. + +Perhaps you should give good old Vedit a try. The Window version 5.0 is due +Jun 2. See +http://www.vedit.com + +Fritz Heberlein + +------------------------------ + +From: G.S.Padhi@soton.ac.uk +Date: Mon, 2 Jun 1997 17:33:31 GMT +Subject: Advanced mathematical typesetting in LaTeX + +Dear Sir, +I was wondering if you know any book on +ADVANCED MATHEMATICAL TYPESETTING in LATEX +Thanks in advance +Padhi + +------------------------------ + +From: Arthur Ogawa for TUG Office <tug-office@tug.cs.umb.edu> +Date: Fri, 25 Jul 1997 12:32:07 +0100 +Subject: TUGboat Volume 18 Issue 1 + +TeX User Group members should soon receive their first TUGboat of 1997, +Volume 18 Number 1. + +If you do not see your expected TUGboat arrive within two weeks +(possibly later for addresses outside the USA), please notify the TeX +User Group by replying to this message. + +If you are not currently a member of TUG and are interested in this +group, please see our home page at http://www.tug.org. The very poplar +TeX Live CD-ROM 2 is now bundled with TUG membership; please see the +link on our home page for more details. + +------------------------------ + +From: Fredy Aquino <fredy@jro.igp.gob.pe> +Date: Tue, 10 Jun 1997 16:56:21 -0400 (EDT) +Subject: SliTeX + +Hi friends!: +I have a question: I need to use the Slitex. + I tried to install it using the disk08 where I could +find the file slitex.zip. I followed the steps indicated to install +a disk : using the 4install.bat. The installation, apparently, was ok +all the files from the diskette were copied to the directory c:\emtex +where I installed the latex. But, when I tried to use it I got a +an error message: +!Latex Error: Unkown option 'errorshow' for package 'tracefnt' +... +1.270 \DeclareSymbolFont + <bold>{OT1}{cmr}{bx} + {n} +Question: What does it mean? + Can you tell where to find the slitex and how do I must install +it so I can use it? +Thank you very much! +Fredy Aquino + +------------------------------ + +From: CyrTUG <cyrtug@cemi.rssi.ru> +Date: Thu, 03 Jul 1997 19:17:58 -0700 +Subject: cyrtug'97 + +***************CyrTUG'97 ANNUAL MEETING IN PETERHOF**************** + in association with + St. Petersburg State University, Math-Mech Faculty + September 8--11, 1997, Peterhof, St. Petersburg +******************************************************************* +Organizing Committee Program Committee +^^^^^^^^^^^^^^^^^^^^ ^^^^^^^^^^^^^^^^^ +Irina Makhovaya (Moscow) Eugenii Pankratiev (Moscow) +John Linn Roseman (New York) Kees van der Laan (Gr\"oningen) +Olga Grineva (St. Petersburg) Alexander Berdnikov (St. Petersburg) +Olga Lapko (Moscow) Joseph Romanovskii (St. Petersburg) +Marina Kuznetsova (Moscow) Stanislav Klimenko (Protvino) + + TeXnical School + ^^^^^^^^^^^^^^^ +1. Reports on hot topics in the use and development of TeX and METAFONT + (multilingual support, electronic publishing, graphics, cyrillic + fonts, etc.) +2. Demonstration of new developments +3. Workshops: Exchange of Experience +4. The TeX Archives +5. New public-domain TeXware +6. Tutorials + +******************************************************************** + Non-TeX Activities + ^^^^^^^^^^^^^^^^^^ +1. An excursion to Oranienbaum +2. An excursion to St. Petersburg +3. An excursion to Peterhof Palace +4. Walks and discussions with colleagues +5. Conference banquet +6. Cultural events +7. A book kiosk + +******************************************************************** + Accommodations + ^^^^^^^^^^^^^^ +1. 1 or 2 persons to a room (WC, shower-bath and so on) +2. Coffee/Tea/Lunch/Dinner +3. A train from St. Petersburg to Peterhof and vice versa on the first + and last days of the meeting, respectively + +******************************************************************* + Application form* + ^^^^^^^^^^^^^^^^ + (Send, please, by e-mail before 31 July, 1997) + +Yes, I want to come to the CyrTUG'97 Annual Meeting! I have +completed the form below. + +Full name (last, first, middle)_____________________________________ + +Member of CyrTUG or TUG or LUG ____________________________________ + +Citizenship: ______________________________________________________ + +Birthday (Day, Year): ________________________________________ + +Passport number: ___________________________________________________ + +Date of issue (Day, Year): __________________________________________ + +Expiration date (Day): _____________________________________________ + +Date of arrival to the hotel (Day): _______________________________ + +Time of arrival (if known) (a.m., p.m.): + +Date of departure from the hotel (Day): + +Type the of room wanted: + double single + +Address: Street_______________________________________________________ + + City________________________State/Province___________________ + + Postal Code_________________Country__________________________ + +Telephone (including international code) + ____________________________________________________________ + +Fax ____________________________________________________________ + +E-mail ____________________________________________________________ + +Do you need visa support? + Yes (if so, make sure you've filled your fax number or snail mail +address) + No + +If yes, please tell us the date of your planned entry into Russia + (Day): _______________________________________________________ + +Date of planned exit from Russia (Day): __________________________ + +Topic of report delivered (if any) _________________________________ + + ____________________________________________________________ + + ____________________________________________________________ + +(Paper submissions and proposals for tutorials should be sent to +Program Connittee: Eugenii Pankratiev, Moscow State University, +e-mail: pankrat@shade.msu.ru. +The deadline for submissions is 31 July 1997) + +Anything you would like to add? ____________________________________ + +____________________________________________________________________ + + Date _________________ Signature _________________ + +_____________________________________________________________________ + +*Participants from foreign (non-C.I.S) countries will pay a +REGISTRATION FEE of US$100.00 and an ACCOMODATION and FOOD fee of +US$150.00--250 (depends on accomodations) in cash on arrival (no checks +or credit cards, please). The discount for TUG or LUG members is +50% of the REGISTRATION FEE +********************************************************************* +For contacts and more information: +Irina Makhovaya +e-mail: cyrtug@mir.msk.su or cyrtug@cemi.rssi.ru +phone: (095) 286-0622 +address: CyrTUG, Mir Publishers, + 2 Pervyi Rizhskii Pereulok, + Moscow, 129820, Russia + +------------------------------ + +From: "K. Berry" <kb@cs.umb.edu> +Date: Sun, 20 Jul 1997 16:59:28 -0400 (EDT) +Subject: TeX directory structure (tds) 0.9993 available + +Another draft (0.9994) of the TeX Directory Structure (TDS) standards document +is now available from: + http://www.tug.org/tds/ + ftp://ftp.tug.org/tex/tds.tar.gz + +The DVI file is: + ftp://ftp.tug.org/tex/tds/tds.dvi + +Soon it will be on CTAN as well, in the `tds' directory. +(finger ctan@mail.tug.org for a list of CTAN hosts.) + +The only differences from 0.9993 are a revised section on AmiWeb2c from +the author and some editorial changes. If no problems arise from the +discussion at the upcoming TUG 97 meeting (http://www.tug.org/tug97/), +this will become version 1.0. + +Please send comments to twg-tds@mail.tug.org. + +kb@mail.tug.org, for the TDS committee + +------------------------------ + +From: Frank Ende <101644.2150@CompuServe.COM> +Date: 22 Jul 97 07:27:41 EDT +Subject: TeX Graphic Package? + +Greetings To Tex UK, +I have just started to use Tex (in it's standard form) ver 3.141. I am +endeavouring to produce fairly complex standard documents(tax forms, +governmental forms etc). + +My question: I am trying to produce fairly exact +documents. As a beginner ( and someone who has spent most +of my adult life in the world of windows) I find TeX difficult to use. Are there +Graphics packages (with a GUI) which produce TeX code? Which ones would you +recommend, and how can I get them quickly (help). Our operating system is SUN OS +5.4 generic sun4m. and we currently using TeX 3.141. + +A second question: We are also trying to run TeX on a Sequent system(DYNIX 4.0 +V4.1.2 i386). We are encountering problems compiling the TeX code. Is there a +pre-compiled executable available somewhere which runs under this system? + +Shane Hoey. + +------------------------------ + +From: Jan Krupa <krupa@alpha.sggw.waw.pl> +Date: Tue, 22 Jul 1997 20:48:13 MET-1MET DST +Subject: boxed, multiline formula + +Could somebody explain (or only suggest) to me how to get the following +"boxed and displayed formula" (I need it to have in book) or may be +there is some macro (LaTeX2e) which does it or may be somebody could send me +some private macro: + _____________________________________________ + | | + | e.g.: \iint_D xydxdy | +(1) | formula 1 \sum.... | + | .... | + | | + | e.g.: \lim.. | + | formula2 .... | +(2) | ..... | + | | + |_____________________________________________| + +or + _____________________________________________ + | | + | e.g.: \iint_D xydxdy | +(1) | formula 1 \sum.... | + | .... | + |_____________________________________________| + | | + | e.g.: \lim.. | + | formula2 .... | +(2) | ..... | + | | + |_____________________________________________| + +I would like to have the box horizontal and vertical centered +and automatically numbered. +I would prefer to have it in LaTeX2e (AMSLaTeX) + +It was told to use minipage environment so +I have tried to use 'minipage' and 'tabular' environments but the results +are rather poor, the places of numbers are not adjusted and the width of +the box is not equal to the width of textwidth (ho to do that?) : + +- ------- +\documentclass[12pt,leqno]{amsart} +\textwidth=12 true cm +\usepackage{amsmath,amssymb} +\begin{document} +bbbbbbb aaaaaaaaa aaaa aaaaaaa aaaaaaaa aaaaaaa aaaaaaa aaaaaa aaaaa +aaaaaa aaaaaa aaaaaaa aaaaaaaa aaaaaa aaaaaaa aaaaaaaaa aaaaaa aaaaaaaa +aaaaaa aaaaaaaa +\par +\bigskip +\noindent +\begin{tabular}{ p{1cm}| p{11cm}|} +\cline{2-2} +(1) +& \begin{minipage}[c]{11 true cm} +\begin{equation} +\iint\limits_D \nonumber +\end{equation} +\end{minipage} \\ +(2) & +\begin{minipage}[c]{11 true cm} +\begin{equation} +\sum_i x_i \nonumber +\end{equation} +\end{minipage} \\ +\cline{2-2} +\end{tabular} +\bigskip + +\begin{minipage}[c]{1cm} +\vspace{-0.8em} +\begin{eqnarray} +\ \\ +\ \nonumber \\ +\ \\ +\ \nonumber \\ +\ \\ +\ \nonumber \\ +\ +\end{eqnarray} +\end{minipage} +\framebox{ +\begin{minipage}{11cm} +%\vspace{-\abovedisplayskip} +\begin{eqnarray} +\iint\limits_D xy dx dy \nonumber\\ +\sum_i x_i \nonumber\\ +\iint\limits_D xy dx dy \nonumber\\ +\sum_{\sum\limits_{k=0}^i}^{\sum\limits_{s=1}^m} x_i \nonumber +\end{eqnarray} +\end{minipage}} + +\end{document} + +- ------ +Thanks in advance. + +Jan Krupa +Warsaw Agricultural University + +------------------------------ + +About TeXhax... + +Please send contributions to: TeXhax@tex.ac.uk + +Subscription and unsubscription requests: + send a one line mail message to TeXhax-Request@tex.ac.uk + containing only the line + subscribe texhax + or + unsubscribe texhax +If you have problems with un/subscribing, +please mail texhax-owner@nottingham.ac.uk + +For information on the TeX Users Group, please send a message to +TUG@TUG.org, or write TeX Users Group, 1850 Union Street, #1637 +San Francisco CA 94123 (phone: 1 415 982 8449, fax: 1 415 982 8559) + +Backnumbers of all the digests are stored in the Comprehensive TeX +Archive Network (CTAN) and can be retrieved on the Internet by +anonymous ftp. The hosts comprising CTAN include, among others, + ftp.dante.de (129.69.1.12) -- Germany + ftp.tex.ac.uk (128.232.1.87) -- UK +Please use your nearest server, to keep network load down. +The file /tex-archive/CTAN.sites on each of these hosts gives a +list of other sites which maintain full or partial mirrors of the CTAN. +Alternatively, finger ctan_us@ftp.shsu.edu for full details. + +TeXhax Digest back issues are filed below /tex-archive/digests/texhax/ +Keyword-In-Context indexes are filed in /tex-archive/digests/indexes/ + +A Hypermail version of TeXhax is also available on the World-Wide Web at URL +http://www.tex.ac.uk/tex-archive/digests/hyper/ + +\bye + +End of TeXhax Digest V1997 #6 +***************************** diff --git a/info/digests/texhax/97/texhax.07 b/info/digests/texhax/97/texhax.07 new file mode 100644 index 0000000000..9dcdb98b8a --- /dev/null +++ b/info/digests/texhax/97/texhax.07 @@ -0,0 +1,232 @@ +Article 74 of ucam.mlist.texhax: +From: owner-texhax-digest@nottingham.ac.UK +Subject: TeXhax Digest V1997 #7 +Date: 28 Aug 97 12:29:48 GMT + +TeXhax Digest Thursday, 28 August 1997 Volume 1997 : Number 007 + +(incorporating UKTeX Digest) + +Today's Topics: + + Re: boxed, multiline formula + greek packages + Making (La)TeX beep + Announcing gsftopk 1.14 + +---------------------------------------------------------------------- + +From: "Carlos A. M. Carvalho" <carlos@fisica.ufpr.br> +Date: Fri, 25 Jul 1997 14:51:15 -0300 +Subject: Re: boxed, multiline formula + +Jan Krupa (krupa@alpha.sggw.waw.pl) wrote on 22 July 1997 20:48: + >Could somebody explain (or only suggest) to me how to get the following + >"boxed and displayed formula" (I need it to have in book) or may be + >there is some macro (LaTeX2e) which does it or may be somebody could send me + >some private macro: + > _____________________________________________ + > | | + > | e.g.: \iint_D xydxdy | + >(1) | formula 1 \sum.... | + > | .... | + > | | + > | e.g.: \lim.. | + > | formula2 .... | + >(2) | ..... | + > | | + > |_____________________________________________| + +I use plain, I hope this helps: + +\def\boxeq#1{\vcenter{\hrule\hbox{\vrule\vbox{\kern 8 pt +\hbox{$\kern 3,5 pt\displaystyle #1\kern 3,5 pt$}\kern 8 pt}\vrule}\hrule} +. + +Usage: $$\boxeq{formula}$$. To get equation numbers you'll have to use +your way. I have my own macros for numbering. It works well for me; +numbers appear outside the box. + +If I understand your drawing correctly, you want numbers on the left +and outside of the box. I don't like this. You can use \eqalignno +inside the box, but the box will include the numbers, which is ugly +but logical. + +Carlos + +------------------------------ + +From: mackay@cs.washington.edu (Pierre MacKay) +Date: Fri, 25 Jul 1997 12:06:25 -0700 +Subject: greek packages + +he Ibycus4 package (the name is a intended as a tribute to David +Packard's Ibycus system, but this package has no connection with +any work done by the Packard Humanities Institute) is available +on osman.classics.washington.edu +[128.95.170.63] in ~ftp/pub/tex, in 4 forms. + +1. iby4str is a streamed SVR4 (Solaris) package, ready for installation + +2, iby4str.gz is the same thing gzipped. There doesn't seem to be any +way to zcat such a file and pipe it into pkgadd. gunzip has to +be used as a separate operation/ + +3. ibycus4.tar.gz is a SVR4 (Solaris) package in spool directory format +if untarred into the directory /var/spool/pkg it is ready for pkgadd + +4. ibycus4.zip is for non-Unix sites. It includes all the genuine files +of Ibycus4 in an 8+3 TDS-conformant style, but not the symbolic links +that make life pleasanter in the Unix world. + +There are also 4 similarly organized files of the Silvio Levy's sources +levystr levystr.gz levygrk.tar.gz levygrk.zip +These contain the unaltered files collected from a CTAN archive in 1994 +reorganized into a TDS-conformant package. + +Further on Ibycus4 + +Changes in setwidths have been made, so spacing will be a bit different +(and better, I hope). Some small improvements in input coding are +made (the 4 distinguishes the new input coding from the old 3 coding). +For other details see the README file. + +Here is the relevant extract from the README file. Ibycus3 is what +was previously known as ibygrk. + + %%%%%%%%%%%%%%%%%%%%%%%%%%%%%%%%%%%%%%%%%%%%%%%%%%%%%%%%%%% + NOTE: THE FOLLOWING CODINGS ARE NOT COMPATIBLE WITH IBYCUS3 + %%%%%%%%%%%%%%%%%%%%%%%%%%%%%%%%%%%%%%%%%%%%%%%%%%%%%%%%%%% + I have tried to keep incompatible codings to the minimum + but the ibycus3 versions of the following were extremely + undesirable. These are all simplifications of ibycus3 coding. + %%%%%%%%%%%%%%%%%%%%%%%%%%%%%%%%%%%%%%%%%%%%%%%%%%%%%%%%%%% + The mark of elision is ' or {'} (the form in braces may be + needed to prevent ' from being read as an accent). + Single quotes may be provided by ` {`} and ' {'}, (isolate them + in braces if necessary). Double quotes are `` {``} and + '' {''} (isolate in braces if necessary). < and > are the + angle brackets used for conjectural supplements. + %%%%%%%%%%%%%%%%%%%%%%%%%%%%%%%%%%%%%%%%%%%%%%%%%%%%%%%%%%% + +The prefix in Karl Berry's font-naming scheme is "fib". The full +naming scheme is provided in the README file. + +%=======================================================================% +| N O T I C E | +| Please note the changes in address and telephone number below. | +| There is no Northwest Computing Support Center any longer. | +| Until further notice, I shall be continuing to provide tape | +| distributions and whatever other services I can. | +| | +%=======================================================================% +Email concerned with UnixTeX distribution software may be sent +To: mackay@cs.washington.edu Pierre A. MacKay +Smail: Department of Classics Emeritus Druid for + Denny Hall, Mail Stop DH-10 Unix-flavored TeX + University of Washington + Seattle, WA 98195 + (206) 543-2268 (Message recorder) + +------------------------------ + +From: "Darren Hill" <PPXDNH@ppn1.physics.nottingham.ac.uk> +Date: Mon, 28 Jul 1997 15:39:49 GMT0BST +Subject: Making (La)TeX beep + +Dear all, + +A small and seemingly simple question, how does one make (La)TeX +beep (ie exude a BEL control code)? I'm currently writing a special +warning (for a \todo command) that will write to screen what I want +to do, and beep to make sure it gets noticed (in much the same way a +LaTeX error does, but without stopping processing). I can do the +screenwriting, but for the life of me I can't find the code that +makes the think beep anywhere in the source! And if I try the method +suggested in the TeXbook all I end up doing is writing variations of +'007 to the screen... + +Any help or working code fragments that anyone could provice would be +gratefully accepted, cos it's driving me nuts! + +Cheers in advance + +Darren + +------------------------------ + +From: vojta@math.berkeley.edu (Paul Vojta) +Date: Fri, 22 Aug 1997 14:25:12 -0700 (PDT) +Subject: Announcing gsftopk 1.14 + +This is to announce that version 1.14 of gsftopk is available. gsftopk is +a utility that calls Ghostscript to render PostScript fonts, and then converts +them into pk format. This allows you to view PostScript fonts in xdvi. + +In addition to various bug fixes, this version: + + 1. Prints out the version of ghostscript being used along with its own + version number. + + 2. Is ported to Windows 95/NT (courtesy of Fabrice Popineau). + + 3. Make some changes to the PostScript environment to accommodate + dotlessj.pro + + 4. Allow psfonts.map lines with several ``specinfo'' strings, such as + + ptmr8r Times-Roman ".167 SlantFont" "TeXBase1Encoding ReEncodeFont" + +The upgraded version is available from CTAN in the file: + + tex-archive/fonts/utilities/gsftopk/gsftopk-1.14.tar.gz + +Recall that CTAN is one of the following hosts: + + tug2.cs.umb.edu + ftp.dante.de + ftp.tex.ac.uk + +or one of their numerous mirror sites. + +- --Paul Vojta, vojta@math.berkeley.edu + +------------------------------ + +About TeXhax... + +Please send contributions to: TeXhax@tex.ac.uk + +Subscription and unsubscription requests: + send a one line mail message to TeXhax-Request@tex.ac.uk + containing only the line + subscribe texhax + or + unsubscribe texhax +If you have problems with un/subscribing, +please mail texhax-owner@nottingham.ac.uk + +For information on the TeX Users Group, please send a message to +TUG@TUG.org, or write TeX Users Group, 1850 Union Street, #1637 +San Francisco CA 94123 (phone: 1 415 982 8449, fax: 1 415 982 8559) + +Backnumbers of all the digests are stored in the Comprehensive TeX +Archive Network (CTAN) and can be retrieved on the Internet by +anonymous ftp. The hosts comprising CTAN include, among others, + ftp.dante.de (129.69.1.12) -- Germany + ftp.tex.ac.uk (128.232.1.87) -- UK +Please use your nearest server, to keep network load down. +The file /tex-archive/CTAN.sites on each of these hosts gives a +list of other sites which maintain full or partial mirrors of the CTAN. +Alternatively, finger ctan_us@ftp.shsu.edu for full details. + +TeXhax Digest back issues are filed below /tex-archive/digests/texhax/ +Keyword-In-Context indexes are filed in /tex-archive/digests/indexes/ + +A Hypermail version of TeXhax is also available on the World-Wide Web at URL +http://www.tex.ac.uk/tex-archive/digests/hyper/ + +\bye + +End of TeXhax Digest V1997 #7 +***************************** diff --git a/info/digests/texhax/97/texhax.08 b/info/digests/texhax/97/texhax.08 new file mode 100644 index 0000000000..d77ecd1687 --- /dev/null +++ b/info/digests/texhax/97/texhax.08 @@ -0,0 +1,258 @@ +Article 75 of ucam.mlist.texhax: +From: owner-texhax-digest@nottingham.ac.UK +Subject: TeXhax Digest V1997 #8 +Date: 19 Sep 97 17:37:58 GMT + +TeXhax Digest Friday, 19 September 1997 Volume 1997 : Number 008 + +(incorporating UKTeX Digest) + +Today's Topics: + + patgen + EuroTeX'98, first Call for Papers + tableofcontents and index problem in AMSLaTeX + Upgrading to LaTeX(2e) + +---------------------------------------------------------------------- + +From: gpg@arbortext.com (Gary Grosso) +Date: Thu, 28 Aug 97 13:24:38 -0400 +Subject: patgen + +Hi all, + +I'm looking for patgen.web. I found a man page, which says: + Frank Liang wrote the first version of this program. Peter + Breitenlohner made a substantial revision in 1991 for TeX 3. + +I would like to download a copy of this latest version. I can't, in fact, +find any version of patgen.web in the TeX archives. Can anyone help me +please? Thanks! + +- -- + +Gary Grosso +ArborText, Inc. +Ann Arbor, MI, USA +gpg@arbortext.com + +------------------------------ + +From: Robin Fairbairns <Robin.Fairbairns@cl.cam.ac.uk> +Date: Wed, 03 Sep 1997 17:01:48 +0100 +Subject: EuroTeX'98, first Call for Papers + + ======================================= +| EuroTeX'98 | +| | +| Xth European TeX Conference | + ======================================= + +Programme Committee +================== + +Thierry BOUCHE -- UJF, Grenoble, France +Andre DESNOYERS -- IBP, Paris, France +Robin FAIRBAIRNS -- Cambridge, UK +Daniel FLIPO -- Maths, Lille, France +Bernard GAULLE -- IDRIS, Paris, France +Michele JOUHET (President) -- CERN, Geneva, Switzerland +Irina A. MAKHOVAYA -- Mir Publishers, Moscow, Russia +Tomasz PRZECHLEWSKI -- University of Gdansk, Poland +Christophe PYTHOUD -- LING, Lausanne, Switzerland +Bernd RAICHLE -- Esslingen, Germany +Philip TAYLOR -- RHBNC, University of London, UK +Gabriel VALIENTE -- Technical University of Catalonia, Catalonia, Spain +Kees VAN DER LAAN -- Garnwerd, The Netherlands +Jiri ZLATUSKA -- Masaryk University, Brno, Czech Republic + +Local Organisation: Jacques Andre (President) -- Irisa, Rennes, France +================== Barbara Auzeby -- Blueprint, St-Germain en Laye, France + Eric Picheral -- CRI Univ. Rennes 1, France + +Conference Chairman: Michel Goossens -- CERN, Geneva, Switzerland + +Call for Papers +============== + +GUTenberg is organizing EuroTeX'98 which will be a part of WEPT'98 the +second Week on Electronic documents and Typography +<http://www.irisa.fr/ep98/week.html> including also the following conferences +EP'98 (Electronic Publishing), RIDT'98 (Raster Imaging and digital +Typography) and PODDP'98 (Principles of Digital Document Processing). +EuroTeX'98 will be the place where the principal actors in the field of +scientific and technical publishing, especially those using TeX as +typesetting engine, will meet. The conference will be held in the corsair +town of St Malo (Brittany, France) from Sunday morning March 29th to +Tuesday evening March 31st 1998. + +Authors are invited to submit proposals for article(s) in English or in +French, which will be reviewed by the Programme Committee in accordance +with the schedule below. The first page should contain the title of the +paper and the full name, affiliation and contact details (including +email address) for each author; the expected time required for oral +presentation should also be specified. + +Calendar +======= +15 October 1997 : Deadline for submission of abstracts. +1 November 1997 : Notification of acceptance or rejection. +15 December 1997 : Deadline for draft version of the papers. +9 January 1998 : Committee response. +1 February 1998 : Deadline for final version of the papers. +29-31 March 1998 : Conference EuroTeX'98. + +An incomplete list of possible subjects follows. +- - Tools, editors, viewers, print drivers for TeX. +- - Specific formats. +- - LaTeX packages. +- - World archives, CTAN servers, maintenance, validation, enhancements. +- - Multi-lingual tools, formats and documents. +- - Fonts. +- - Standardization. +- - Multimedia publications. +- - Page models, style sheets, DSSL, ... +- - PostScript, PDF, SGML, HTML, XML, MathML applications. +- - Graphics, sound and pictures. +- - Editorial chain. +- - Internet explorers. +- - Editorial, bibliographic and textual databases. +- - Revisable documents. +- - Copyright questions with respect to scientific and technical + publications. +- - Journalism, publicity, works of art, printing. +- - Scientific publications: tools for mathematics, physics, chemistry, + etc. +- - What future for paper publication versus electronic publication? +- - Possibilities offered by the new digital media: CD-ROM, Zip, etc. +- - Publication and consultation at home or in the office. +- - Conversion of proprietary formats. +- - LaTeX versus other products from competitors. + +Proposals should be transferred using FTP to our server as follows: +ftp cadd.cern.ch +user anonymous +password your e-mail address +cd /pub/cern/DTPs/Eurotex + +then create an author-specific directory. After having copied their +file(s) to this directory the authors are requested to send a message to +the Chair of the Programme Committee advising her that the transfer has +taken place (michele.jouhet@cern.ch). + +The authors whose articles are accepted will receive instructions for +the preparation of the final version of their paper. Accepted papers +will be published in the conference proceedings, which will be available +at the conference. + +For further information in English or French, please write to: +Michele JOUHET -- CERN/AS, 1211 Geneve 23, Suisse +(michele.jouhet@cern.ch) +Bernard GAULLE -- IDRIS, France (gaulle@idris.fr) + +------------------------------ + +From: Jan Krupa <krupa@alpha.sggw.waw.pl> +Date: Mon, 08 Sep 1997 15:31:18 MET-1MET DST +Subject: tableofcontents and index problem in AMSLaTeX + +Dear La(TeX) users, + +I use AMSLaTeX (\documentclass[11pt]{amsbook}) and have problem with +** tableofcontents **. I mean: +- - there is no hyphenation when the titles of chapters or (sub)section + are long. e.g.: + + CONTENTS + .... + 3.2.4 The nucleus of thermoelastic strain in a space with a spherical cavity, + in a solid sphere and in a shell 30 + +but there should be + + 3.2.4 The nucleus of thermoelastic strain in a space with a spheri- + cal cavity, in a solid sphere and in a shell.................. 30 + +** Similar things happen when try to generate index ** + +- - there is no the "dots" like below + 3.2.1 Some title .................................................... 20 + +I have + + 3.2.1 Some title 20 + +instead. + +Could someone suggest what is going on ? + +When I use the 'book' class instead 'amsbook' the tableofcontents +and 'index' are nice but I would +prefer to use amsbook. + +If would be so kind please send the answer to 'krupa@alpha.sggw.waw.pl' +because I have often trouble to access to the news so I could miss the +answer. + +Best Regards + +Jan Krupa + +------------------------------ + +From: Edward L Chupp <elc@hopper.unh.edu> +Date: Wed, 10 Sep 1997 09:25:33 -0400 (EDT) +Subject: Upgrading to LaTeX(2e) + +In the late 1980's I bought a TeX package (version 2.09). I now need an +upgrade to LaTeX(2e). I specifically need the style file epsfig.sty, but +upon examining it I noted that it needs a number of other files aswell. +Could you please advise me where I can obtain this package (LaTeX(2e) +either commercially or by shareware. I am not a TeXpert, so I would not +know hot to "upgrade," i. e., I probably would have to start with a brand +new TeX/LaTeX. + +Thank you for your help! + +- --mary:) + +------------------------------ + +About TeXhax... + +Please send contributions to: TeXhax@tex.ac.uk + +Subscription and unsubscription requests: + send a one line mail message to TeXhax-Request@tex.ac.uk + containing only the line + subscribe texhax + or + unsubscribe texhax +If you have problems with un/subscribing, +please mail texhax-owner@nottingham.ac.uk + +For information on the TeX Users Group, please send a message to +TUG@mail.tug.org, or write TeX Users Group, P.O. Box 1239, Three Rivers, +CA 93271-1239 (phone: 1 209 561 0112, fax: 1 209 561 4584) + +Backnumbers of all the digests are stored in the Comprehensive TeX +Archive Network (CTAN) and can be retrieved on the Internet by +anonymous ftp. The hosts comprising CTAN include, among others, + ftp.dante.de (129.69.1.12) -- Germany + ftp.tex.ac.uk (128.232.1.87) -- UK +Please use your nearest server, to keep network load down. +The file /tex-archive/CTAN.sites on each of these hosts gives a +list of other sites which maintain full or partial mirrors of the CTAN. +Alternatively, finger ctan_us@ftp.shsu.edu for full details. + +TeXhax Digest back issues are filed below /tex-archive/digests/texhax/ +Keyword-In-Context indexes are filed in /tex-archive/digests/indexes/ + +A Hypermail version of TeXhax is also available on the World-Wide Web at URL +http://www.tex.ac.uk/tex-archive/digests/hyper/ + +\bye + +End of TeXhax Digest V1997 #8 +***************************** diff --git a/info/digests/texhax/97/texhax.09 b/info/digests/texhax/97/texhax.09 new file mode 100644 index 0000000000..659373289b --- /dev/null +++ b/info/digests/texhax/97/texhax.09 @@ -0,0 +1,431 @@ +Article 76 of ucam.mlist.texhax: +From: owner-texhax-digest@nottingham.ac.UK +Subject: TeXhax Digest V1997 #9 +Date: 14 Oct 97 10:29:50 GMT + +TeXhax Digest Tuesday, 14 October 1997 Volume 1997 : Number 009 + +(incorporating UKTeX Digest) + +Today's Topics: + + Re: patgen + LaTeX2e June 1997 + Getting the Index to Work - No Flames Please + Re: Commercially available LaTeX for SGI? + New releases of changebar and supertabular + Q booch diagram typesetting + LaTeX2e question + BiBTeX question + Announcing gsftopk 1.15 + release of AMSFonts in PostScript format + Re: release of AMSFonts in PostScript format + Re: release of AMSFonts in PostScript format + +---------------------------------------------------------------------- + +From: "K. Berry" <kb@cs.umb.edu> +Date: Mon, 22 Sep 1997 17:49:37 -0400 (EDT) +Subject: Re: patgen + + find any version of patgen.web in the TeX archives. Can anyone help me + +It's at many places. It's by itself at: +systems/knuth/unsupported/texware/patgen.web +on CTAN (e.g., tug2.cs.umb.edu:/tex-archive ...) + +------------------------------ + +From: "SIDNEY J. Katzen" <SJ.Katzen@ulst.ac.uk> +Date: Tue, 23 Sep 1997 09:14:05 GMT +Subject: LaTeX2e June 1997 + +In older versions of LaTeX2e I was able to alter the textwidth of the +document in a homemade style file read at the beginning before +\begin{document} in the normal way: +\setlength{\textwidth}{etc}. +Now I find that that this has to be inserted before the +\begin{document} to have any effect. Alternatively I can use +\AtBeginDocument. Other page dimension alterations in this style +file seem to work OK. Is this a bug? + +Sid K +Dr. S.J. Katzen +School of Electrical and Mechanical Engineering (5F06) +University of Ulster +Jordanstown +County Antrim +N. Ireland +UK BT37 0QB + +Phone: X-44-1232-36-6448 +Fax: X-44-1232-36-6804 +e-mail: sj.katzen@ulst.ac.uk +URL: http://www.nibec.ulst.ac.uk/~sidk + +------------------------------ + +From: wittenm@umich.edu (Matthew Witten) +Date: Wed, 24 Sep 1997 00:18:42 -0400 +Subject: Getting the Index to Work - No Flames Please + +This is probably a stupid question. But I am a Tex person trying to learn +how to use latex and makeindex (unix system). I am trying to find the +command to actually generate the final index and understand that if your +file containing the index details is foo.idx, then the command to make the +new printable index is + +makeindex foo.idx + +Unfortunately, when I use the command makeindex, or any variant, I get the +error cannot find command makeindex. Can someone help me out here? I'm not +a novice at tex, just at latex and making indexes. And, while your at it, +anyone with glossary development experience - where is the makeglossary +command hidden? + +Please send comments directly to me. I will summarize for the list if there +is interest. Thanks for your patience. + +Tarynn M. Witten, Ph.D. + +------------------------------ + +From: Paul Garlick <itw-devilbiss.design@thenet.co.uk> +Date: Thu, 25 Sep 1997 09:06:52 +0100 +Subject: Re: Commercially available LaTeX for SGI? + +Dear User Group, + +May I ask a quick question on the availability of a commercial version of +LaTeX to run on the workstation I am using. It is a Silicon Graphics Indigo +2 R10000 running IRIX 6.2 (without a direct connection to the internet at +present). Following a tip from the UK-TUG FAQ list I have contacted Arbor Text +Inc. but they in fact no longer supply the software. Do you have any +information on alternatives? or would I be better off buying a modem and +downloading the free version? The latter approach seems attractive but I do +not know any TeX gurus to ask if I get stuck. If I could find the price of +a commercial version I could weigh up the pros and cons. + +Many thanks in advance for your help, + +Paul Garlick (imminent MSc thesis writer!). + +------------------------------ + +From: "Johannes L. Braams" <jlbraams@cistron.nl> +Date: Sun, 28 Sep 1997 20:13:04 +0200 +Subject: New releases of changebar and supertabular + +Hello, + +As of tomorrow 29th september both the packages supertabular and changebar +will be available from CTAN in new versions, as described below. + +Supertabular version 4.1a: +Two new environments have been added, mpsupertabular and mpsupertabular*. +They were provided by Axel Kielhorn <A.Kielhorn@tu-bs.de>. These environemnts +are variants of the old supertabular environments that put each tabular +segment in its own minipage. This easses the addition of footnotes to +a multipage tabular. +Fixed a few bugs, one with \\[...] inside \tablehead and \tabletail, the +other in the calculation of the available height on the first page of the +supertabular. This one affected it's use in twocolumn mode only. + +Changebar version 3.3c +Finally fixed the bug that produced diagonal bars when a changebar was +started near the end of a page. Also made changebar work in LaTeX's +twocolumn mode, in twocolumn the bars appear on the `outside' of each +column (which means to the left of the left column and to the right of +the right column). +Also fixed a bug in the calculation of the positioning of the bars. + +If you encounter any problems with these packages please report them +to me, using the latexbug.tex mechanism + +Kind regards, + +Johannes Braams + +------------------------------ + +From: Aleksei Makarov <makarov@iitam.omsk.net.ru> +Date: Thu, 2 Oct 1997 16:13:05 +0000 ( ) +Subject: Q booch diagram typesetting + +Hi! + +Is there a pakage that simplifies the typesetting of booch diagrams +(those as used in object-oriented porgramming)? + +A. Makarov + +------------------------------ + +From: "AS Dawes" <adawes@awe.co.uk> +Date: Fri, 3 Oct 1997 13:09:18 +0100 +Subject: LaTeX2e question + +Can you please tell me if there is a package for LaTeX2e +which I can get hold of on the WWW that does the following +or similar:- + +I have a postscript file in EPS format which I want to +place ANYWAY on my page. The implication of this is that +it can overwrite text etc if present under it. I wish to +set properties such as its height and width as well as +an arbitrary origin on the page (for example coordinates +could be measured from the bottom left hand corner of page). +I hope you can help and look forward to hearing from you. + +Thank you + +Regards +Dr. Alan Dawes + +p.s. I am using LaTeX2e <1996/06/01> + +------------------------------ + +From: Tomasz GRZEGORCZYK <grzegor@lemahp6.epfl.ch> +Date: Tue, 07 Oct 1997 17:19:14 METDST +Subject: BiBTeX question + +Since it is the first time I write to this address I do +not know how is it organized, who am I speaking to +and how should I write this e-mail. +So, sorry for the `non personnal' mail and I hope to +write it in a better way next time! + +The question I have is about BiBTeX and cross references. +As a matter of fact, it is easy to make them by number but +is it also possible (and how!) to make them by name and date? +I mean that so far, the output in the text (article, PhD thesis) is +a reference like `[number]' and one has to go to the bibliography to +check what does it correspond to. Instead of that, I would like to +have in the text something like `[name, date]'. +Does this format already exist or should I try to write a macro +by myself? + +Thank you for your help! +Tomasz + +PS: the LaTeX version I am using is Version 3.14159 (C version 6.1), +LaTeX2e <1996/06/01> + +- -- +- --------------------------------------- +Tomasz Grzegorczyk +Grzegorczyk@lema.epfl.ch +Swiss Federal Institute of Technology, Lausanne +LEMA-DE-EPFL +CH-1015 Lausanne +Switzerland +phone: +41-21-693 4643 +fax: +41-21-693 2673 +http://lemawww.epfl.ch/ +- --------------------------------------- + +------------------------------ + +From: vojta@math.berkeley.edu (Paul Vojta) +Date: Thu, 9 Oct 1997 12:01:23 -0700 (PDT) +Subject: Announcing gsftopk 1.15 + +This is to announce that version 1.15 of gsftopk is available. gsftopk is +a utility that calls Ghostscript to render PostScript fonts, and then converts +them into pk format. This allows you to view PostScript fonts in xdvi. + +In addition to various bug fixes, this version: + + 1. Is ported to the Amiga (courtesy of Christophe Labouisse). + + 2. Accepts '<<', '<[', and '<<[' syntax in the psfonts.map file. + + 3. Accepts numerous additional command-line arguments. + + 4. Has better configuration scripts (thanks to Nelson Beebe for help + on this one). + +The upgraded version is available from CTAN in the file: + + tex-archive/fonts/utilities/gsftopk/gsftopk-1.15.tar.gz + +Recall that CTAN is one of the following hosts: + + tug2.cs.umb.edu + ftp.dante.de + ftp.tex.ac.uk + +or one of their numerous mirror sites. + +- --Paul Vojta, vojta@math.berkeley.edu + +------------------------------ + +From: Ralph Youngen <rey@ams.org> +Date: Thu, 09 Oct 1997 23:12:58 -0400 (EDT) +Subject: release of AMSFonts in PostScript format + +The American Math Society is pleased to announce the release, for free +and unrestricted use, the Type-1 (PostScript) versions of the AMSFonts. +This collection includes the Euler, ``extra symbol'' and cyrillic fonts +in selected sizes. + +These fonts, produced by Blue Sky Research, of Portland, Oregon, and +Y&Y Inc., of Concord, Massachusetts, and previously distributed by them, +are made available through the cooperation of a consortium of scientific +publishers along with Blue Sky and Y&Y. Members of this consortium include: + + Elsevier Science + IBM Corporation + Society for Industrial and Applied Mathematics (SIAM) + Springer-Verlag + American Mathematical Society (AMS) + +In order to assure the authenticity of these fonts, copyright will be +held by the AMS. This is not meant to restrict in any way the legitimate +use of the fonts. However, the AMS does require that the AMS copyright +notice be removed from any derivative versions of the fonts which have +been altered in any way. A more detailed statement of these requirements +can be found at the AMS Web site, http://www.ams.org/tex/ . + +The canonical versions of these fonts are located on the AMS FTP server +and are also accessible via the Web: + + ftp://ftp.ams.org/pub/tex/psfonts/ams + +Four system-specific packages are available from the AMS site: + + for use on Macintosh, containing the fonts in standard Macintosh + Type 1 format + + - packaged as required for use by Textures + - packaged for use by OzTeX, CMacTeX, or other Mac TeX implementations + + for use on a Windows or DOS system, containing fonts in PFB format with + PFM metrics files; + + for use on a Unix system, containing fonts in PFB format with AFM + metrics files. + +Each system-specific package is accompanied by a READ.ME file which contains +instructions for downloading and installing the fonts. Please review the +appropriate READ.ME file in its entirety before undertaking to install the +fonts on your system. + +The fonts in Macintosh and PFB (binary Type 1 outline) formats, along with +metrics in appropriate form, will be mirrored on the Comprehensive TeX +Archive Network (CTAN) in the area + + fonts/amsfonts/ps-type1 + +Users requiring the fonts in PFA (ASCII Type 1) form should convert them +with the aid of one of the tools available for that purpose from CTAN. + +The AMS does not provide installation assistance or technical support +beyond any installation instructions included with the collection. +Installation and use of these fonts may require some technical expertise. + +------------------------------ + +From: ralph@pierce.math.hawaii.edu (Ralph Freese) +Date: Fri, 10 Oct 1997 14:27:49 -1000 +Subject: Re: release of AMSFonts in PostScript format + +Great! Thanks!! + + Ralph + +One slight glitch in how these fonts are set up on TeXLive2 which +may effect how TeXLive3 is set up: + +We copied the ams pfb files into a new directory under +./texmf/fonts/type1 and changed dvips's map file for the +cmps printer to use these rather than the bakoma versions. +The glitch is that bokoma had some intermediate sizes the ams +didn,t, for example, msbm8. Since mxbm8.tfm exists, the dvi file +can have it and then using + + dvips file.dvi -P cmps -o + +will fail because there is no msbm8.pfb in the ams map file. + +Of course this can be solved in several ways like getting rid of +msbm8.tfm. What we did was make the file ../texmf/dvips/cmps/config.cmps +be + + p +amsbkm.map + p +cmbkm.map + p +cmps.map + p +amsps.map + +(amsps.map is the ams's map file.) In this case the ps file generated +will have the fonts like MSBM10 (from the ams) and msbm8 (from bakoma). + +------------------------------ + +From: ralph@pierce.math.hawaii.edu (Ralph Freese) +Date: Sun, 12 Oct 1997 13:30:33 -1000 +Subject: Re: release of AMSFonts in PostScript format + +Thanks. That's certainly the answer. The unix distribution of the amspsfonts +didn't mention this but it is in the documentation for the amsfonts. + +But one more glitch: using this from unix gives + +! LaTeX Error: File `OMXcmex.fd' not found. + +As I know you know, this is because LaTeX has changed to all lower case +for these files. For now I'll link OMXcmex.fp to omxcmes.fd, but the +AMS will have to upgrade its amsfonts.sty file. + + Thanks again, + Ralph + +> Hi: That is what the psamsfonts option is for! +> +> Use it when you load the AMS package in LaTeX 2e. +> +> -- +> Berthold K.P. Horn mailto:bkph@ai.mit.edu +> Cambridge, Massachusetts, USA + +------------------------------ + +About TeXhax... + +Please send contributions to: TeXhax@tex.ac.uk + +Subscription and unsubscription requests: + send a one line mail message to TeXhax-Request@tex.ac.uk + containing only the line + subscribe texhax + or + unsubscribe texhax +If you have problems with un/subscribing, +please mail texhax-owner@nottingham.ac.uk + +For information on the TeX Users Group, please send a message to +TUG@mail.tug.org, or write TeX Users Group, P.O. Box 1239, Three Rivers, +CA 93271-1239 (phone: 1 209 561 0112, fax: 1 209 561 4584) + +Backnumbers of all the digests are stored in the Comprehensive TeX +Archive Network (CTAN) and can be retrieved on the Internet by +anonymous ftp. The hosts comprising CTAN include, among others, + ftp.dante.de (129.69.1.12) -- Germany + ftp.tex.ac.uk (128.232.1.87) -- UK +Please use your nearest server, to keep network load down. +The file /tex-archive/CTAN.sites on each of these hosts gives a +list of other sites which maintain full or partial mirrors of the CTAN. +Alternatively, finger ctan_us@ftp.shsu.edu for full details. + +TeXhax Digest back issues are filed below /tex-archive/digests/texhax/ +Keyword-In-Context indexes are filed in /tex-archive/digests/indexes/ + +A Hypermail version of TeXhax is also available on the World-Wide Web at URL +http://www.tex.ac.uk/tex-archive/digests/hyper/ + +\bye + +End of TeXhax Digest V1997 #9 +***************************** diff --git a/info/digests/texhax/97/texhax.10 b/info/digests/texhax/97/texhax.10 new file mode 100644 index 0000000000..c6bb0c03c0 --- /dev/null +++ b/info/digests/texhax/97/texhax.10 @@ -0,0 +1,168 @@ +Article 77 of ucam.mlist.texhax: +From: owner-texhax-digest@nottingham.ac.UK +Subject: TeXhax Digest V1997 #10 +Date: 4 Nov 97 10:31:00 GMT + +TeXhax Digest Tuesday, 4 November 1997 Volume 1997 : Number 010 + +(incorporating UKTeX Digest) + +Today's Topics: + + seminar.cls + [none] + EuroTeX'98: last Call for Papers + Re: Commercially available LaTeX for SGI? + +---------------------------------------------------------------------- + +From: pjbk@cee.hw.ac.uk (Peter JB King) +Date: Tue, 14 Oct 1997 15:12:22 +0100 (BST) +Subject: seminar.cls + +I recently retrieved seminar.cls from twx.ac.uk, having observed that +it had been updated. The version there, seems to have an extraneous } +in it, causing error messages from LaTeX. As far as I recall this was +immediately before the % end change of DPC 1997/09/16 + +The version info is + %% BEGIN seminar.cls + %% LaTeX2e seminar class, derived by Sebastian Rahtz from + %% `seminar.sty', generated on <1993/10/8>. Fixes from David Carlisle + %% and Denis Girou. + %% + %\def\fileversion{1.0} + %\def\filedate{1993/04/01} + \def\fileversion{1.2} + \def\filedate{1997/09/16} + %% + +I had also supposed that it might have been modified to conform to its +documentation, which states that a4 or a4paper is acceptable in the +\documentclass optional argument. In fact, only a4 is acceptable, +a4paper causing an error. + (/usr/lib/teTeX/texmf/tex/latex/seminar/seminar.cls + Document Class: seminar 1997/09/16, 1.2 + Document Style: `seminar' v1.2 <1997/09/16> (tvz) + ! Undefined control sequence. + \ds@a4paper ->\ds@a + 4 + l.91 \ifnotesonly + \else\notesonlystarfalse\fi + +- -- +Peter King, Computing & Electrical Eng. Internet: pjbk@cee.hw.ac.uk + Heriot-Watt University, Riccarton, or P.J.B.King@heriot-watt.ac.uk + Edinburgh EH14 4AS, Scotland +Phone: (+44) 131 451 3433 Fax: (+44) 131 451 3431 + +------------------------------ + +From: piyas@juphys.ernet.in +Date: Fri, 17 Oct 1997 17:03:59 +0500 (GMT+0500) +Subject: [none] + +I am a Ph.D student. I'm very much interested to develop my expertise in +developing LATEX MAcros. Would you please help me in this regard sending +a helping manual. If the manual is available via e-mail, please send me +the file in compressed form. + +Regards. +Piyas Samanta + +P.S PLEASE CORRESPOND ME IN THE FOLLOWING ADDRESS + + ------------------------------------------------------- + Piyas Samanta + c/o Dr. C. K. Sarkar + Dept. of Electronics & Tele-Comm. Engg. + Jadavpur University + CALCUTTA 700 032, INDIA + + E-mail: piyas@juphys.ernet.in Fax: +91 33 473 2217 + ------------------------------------------------------ + +------------------------------ + +From: "Philip Taylor (RHBNC) <Qhaa006@Vms.Rhbnc.Ac.Uk>" <QHAA006@vms.rhbnc.ac.uk> +Date: Tue, 28 Oct 1997 12:30:04 GMT +Subject: EuroTeX'98: last Call for Papers + +Final Call for Papers: the 1998 EuroTeX Conference at St Malo, France. + +Dear Colleague -- apologies if you receive multiple copies of this message, +but time is short and we must circulate as many TeX lists as possible before +the Call for Papers closes. Please see + + http://www.ens.fr/gut/manif/eurotex98/ + +for the official announcement of EuroTeX'98; the Call for Papers closes on +Monday 3rd November 1997. + +Philip Taylor, for the EuroTeX'98 Programme Committee. + +------------------------------ + +From: nospam@oddhack.engr.sgi.com (Jon Leech) +Date: Thu, 30 Oct 1997 19:03:26 -0800 (PST) +Subject: Re: Commercially available LaTeX for SGI? + +Paul Garlick writes: +> May I ask a quick question on the availability of a commercial version of +> LaTeX to run on the workstation I am using. It is a Silicon Graphics Indigo +> 2 R10000 running IRIX 6.2 (without a direct connection to the internet at +> present). Following a tip from the UK-TUG FAQ list I have contacted Arbor Text +> Inc. but they in fact no longer supply the software. Do you have any +> information on alternatives? + + I don't know about commercial versions for Irix. However, you can +just install the fw_tex package from the Irix 6.2 "Freeware 2.0" CD. This +includes TeX, LaTeX, documentation, binaries, etc. so you're pretty much set +to go. + + I'm no TeXpert, but I am successfully using this LaTeX to build the +OpenGL API specification - a 350 page document of complexity comparable to +the typical Ph.D. thesis. Hopefully it will do the job for you too. + + Jon Leech + Silicon Graphics + +------------------------------ + +About TeXhax... + +Please send contributions to: TeXhax@tex.ac.uk + +Subscription and unsubscription requests: + send a one line mail message to TeXhax-Request@tex.ac.uk + containing only the line + subscribe texhax + or + unsubscribe texhax +If you have problems with un/subscribing, +please mail texhax-owner@nottingham.ac.uk + +For information on the TeX Users Group, please send a message to +TUG@mail.tug.org, or write TeX Users Group, P.O. Box 1239, Three Rivers, +CA 93271-1239 (phone: 1 209 561 0112, fax: 1 209 561 4584) + +Backnumbers of all the digests are stored in the Comprehensive TeX +Archive Network (CTAN) and can be retrieved on the Internet by +anonymous ftp. The hosts comprising CTAN include, among others, + ftp.dante.de (129.69.1.12) -- Germany + ftp.tex.ac.uk (128.232.1.87) -- UK +Please use your nearest server, to keep network load down. +The file /tex-archive/CTAN.sites on each of these hosts gives a +list of other sites which maintain full or partial mirrors of the CTAN. +Alternatively, finger ctan_us@ftp.shsu.edu for full details. + +TeXhax Digest back issues are filed below /tex-archive/digests/texhax/ +Keyword-In-Context indexes are filed in /tex-archive/digests/indexes/ + +A Hypermail version of TeXhax is also available on the World-Wide Web at URL +http://www.tex.ac.uk/tex-archive/digests/hyper/ + +\bye + +End of TeXhax Digest V1997 #10 +****************************** diff --git a/info/digests/texhax/97/texhax.11 b/info/digests/texhax/97/texhax.11 new file mode 100644 index 0000000000..bf014fdd74 --- /dev/null +++ b/info/digests/texhax/97/texhax.11 @@ -0,0 +1,366 @@ +Article 78 of ucam.mlist.texhax: +From: owner-texhax-digest@nottingham.ac.UK +Subject: TeXhax Digest V1997 #11 +Date: 21 Dec 97 17:08:04 GMT + +TeXhax Digest Sunday, 21 December 1997 Volume 1997 : Number 011 + +(incorporating UKTeX Digest) + +Today's Topics: + + Web2c-7.1 + ANNOUNCE: xtem version 6 + Harvard Refs? + Harvard Refs? (A) + LaTeX2e version in color.sty + +---------------------------------------------------------------------- + +From: Olaf Weber <infovore@xs4all.nl> +Date: 10 Nov 1997 22:21:01 +0100 +Subject: Web2c-7.1 + +(EST)" +Precedence: bulk + +At long last web2c-7.1 is available. This release contains a port of the +basic TeX project programs (TeX, Metafont, MetaPost, GFtoPK, etc.) to +Unix. It also contains a new version of the kpathsea library. It does not +contain any of the dvi drivers (dviljk, dvipsk, xdvik) or other support +programs (makeindexk, ps2pkk): these will become available later. + +The release can be obtained by retrieving + + ftp://ftp.tug.org/tex/web2c/web2c.tar.gz + ftp://ftp.tug.org/tex/web2c/web.tar.gz + +(the latter is only needed if you do not have the web files, otherwise +you can just link/copy those in the web2c-7.1/web2c directory). You +will also need a a library tree to compile and use these programs. +The file + + ftp://ftp.tug.org/tex/web2c/texmflib.tar.gz + +provides a bare-bones tree. These files are also available as + + ftp://ftp.cs.umb.edu/pub/tex/web2c.tar.gz + ftp://ftp.cs.umb.edu/pub/tex/web.tar.gz + ftp://ftp.cs.umb.edu/pub/tex/texmflib.tar.gz + +Soon these files will also be available on CTAN sites and their +mirrors. + + (Germany) ftp://ftp.dante.de/tex-archive/systems/web2c + (England) ftp://ftp.tex.ac.uk/tex-archive/systems/web2c + (USA) ftp://tug2.cs.umb.edu/tex-archive/systems/web2c + +Finger ctan@ftp.dante.de to obtain a list of mirrors. An up-to-date list +is also appended to this message. + +Please note that the all-in-one-file texk.tar.gz release has _not_ +been updated, and will not be in the immediate future. Sorry. + +This release fixes virtually all known bugs in web2c-7.0 and kpathsea-3.0, +and adds a few new features, especially to kpathsea. Many thanks to all +the people who reported bugs, made suggestions, and provided patches. + +Instructions for sending bug reports (and joining the mailing list) are in +the distribution file kpathsea/BUGS. + +A summary of the changes follows; read them carefully. Please read +the manuals as well. + +kpathsea changes: 3.1 (10 november 1997) +* The support for multiple texmf trees is much improved. +* New MakeTeX* scripts, based on teTeX's. These scripts now use + heuristics to decide where generated files are placed and which + permissions they should have. +* The varfonts feature is enabled by default. +* Some MSDOS/Win32 support added to the sources. +* Change in searching algorithm: if a name doesn't have the default suffix + we append it for the first search. Then we search for the bare name. + Thus if you have foo.bar and foo.bar.tex, the latter will now be found + first. Under the old system, you had to specify boo.bar.tex.tex to get + foo.bar.tex instead of foo.bar. +* Prepend the value of KPSE_DOT to relative paths from texmf.cnf if it + defined in the environment. This means we can use . instead of + $KPSE_DOT in paths. +* Define brace expansion so that {a,b}{1,2} expands to a1:b1:a2:b2. This + is different from how shells do it, and exploited in texmf.in. +* Renamed texmf.cnf.in to texmf.in. +* New value for debugging: DEBUG_VARS, equal to 64. +* If a file format has no suffix, allow its long name as the argument to + kpsewhich --format. + +web2c changes: 7.1 (10 november 1997) +* Some MSDOS/Win32 support added to the sources. +* VPtoVF: Increased capacity. +* TeX, METAFONT, MetaPost: + - File foo.bar.tex results in \jobname foo.bar, hence foo.bar.{log,dvi,...}. + This as opposed to foo and foo.{log,dvi,...} which it how it used to be in + version 6. In 7.0 situation was confusing, as the heuristics used would + sometimes result in the extension being stripped, sometimes not. +* TeX: + - The -extend-jobname option has been removed. + - Printing of non-ASCII characters in diagnostics, code by Bernd Raichle. +* web2c: + - Some support for e-TeX added to convert. + +. +$ finger ctan@ftp.dante.de +[sun.dante.de] +Login name: ctan +Directory: /home/server/ctan Shell: /bin/sync +Never logged in. +No unread mail +Plan: +In order to reduce network load, it is recommended that you use the +Comprehensive TeX Archive Network (CTAN) host which is located in the +closest network proximity to your site. Alternatively, you may wish to +obtain a copy of the CTAN via CD-ROM (see help/CTAN.cdrom for details). + +Known mirrors of the CTAN reside on (alphabetically): + cis.utovrm.it (Italia) /TeX + ctan.math.mun.ca (Newfoundland, Canada) /tex-archive + ctan.unsw.edu.au (NSW, Australia) /tex-archive + dongpo.math.ncu.edu.tw (Taiwan) /tex-archive + ftp.belnet.be (Belgium) /packages/TeX + ftp.ccu.edu.tw (Taiwan) /pub/tex + ftp.cdrom.com (West coast, USA) /pub/tex/ctan + ftp.comp.hkbu.edu.hk (Hong Kong) /pub/TeX/CTAN + ftp.cs.rmit.edu.au (Australia) /tex-archive + ftp.cs.ruu.nl (The Netherlands) /pub/tex-archive + ftp.cstug.cz (The Czech Republic) /pub/tex/CTAN + ftp.duke.edu (North Carolina, USA) /tex-archive + ftp.funet.fi (Finland) /pub/TeX/CTAN + ftp.gwdg.de (Deutschland) /pub/dante + ftp.hea.ie (Ireland) /pub/ctan/tex + ftp.jussieu.fr (France) /pub4/TeX/CTAN + ftp.kreonet.re.kr (Korea) /pub/CTAN + ftp.loria.fr (France) /pub/unix/tex/ctan + ftp.mpi-sb.mpg.de (Deutschland) /pub/tex/mirror/ftp.dante.de + ftp.nada.kth.se (Sweden) /pub/tex/ctan-mirror + ftp.oleane.net (France) /pub/mirrors/CTAN/ + ftp.rediris.es (Espa\~na) /mirror/tex-archive + ftp.rge.com (New York, USA) /pub/tex + ftp.riken.go.jp (Japan) /pub/tex-archive + ftp.tu-chemnitz.de (Deutschland) /pub/tex + ftp.u-aizu.ac.jp (Japan) /pub/tex/CTAN + ftp.uni-augsburg.de (Deutschland) /tex-archive + ftp.uni-bielefeld.de (Deutschland) /pub/tex + ftp.unina.it (Italia) /pub/TeX + ftp.uni-stuttgart.de (Deutschland) /tex-archive (/pub/tex) + ftp.univie.ac.at (\"Osterreich) /packages/tex + ftp.ut.ee (Estonia) /tex-archive + ftpserver.nus.sg (Singapore) /pub/zi/TeX + joshua.smcvt.edu (Vermont, USA) /pub/tex + src.doc.ic.ac.uk (England) /packages/tex/uk-tex + sunsite.auc.dk (Denmark) /pub/tex/ctan + sunsite.cnlab-switch.ch (Switzerland) /mirror/tex + sunsite.icm.edu.pl (Poland) /pub/CTAN + sunsite.unc.edu (North Carolina, USA) /pub/packages/TeX + wuarchive.wustl.edu (Missouri, USA) /packages/TeX + +Known partial mirrors of the CTAN reside on (alphabetically): + ftp.adfa.oz.au (Australia) /pub/tex/ctan + ftp.fcu.edu.tw (Taiwan) /pub2/tex + ftp.germany.eu.net (Deutschland) /pub/packages/TeX + ftp.gust.org.pl (Poland) /pub/TeX + ftp.jaist.ac.jp (Japan) /pub/TeX/tex-archive + ftp.uu.net (Virginia, USA) /pub/text-processing/TeX + sunsite.dsi.unimi.it (Italia) /pub/TeX + sunsite.snu.ac.kr (Korea) /shortcut/CTAN + +Please send updates to this list to <ctan@urz.uni-heidelberg.de>. + +The participating hosts in the Comprehensive TeX Archive Network are: + ftp.dante.de (Deutschland) + -- anonymous ftp /tex-archive (/pub/tex /pub/archive) + -- gopher on node gopher.dante.de + -- email via ftpmail@dante.de + -- World Wide Web access on http://www.dante.de/ + -- Administrator: <ftpmaint@dante.de> + + ftp.tex.ac.uk (England) + -- anonymous ftp /tex-archive (/pub/tex /pub/archive) + -- gopher on node gopher.tex.ac.uk + -- NFS mountable from nfs.tex.ac.uk:/public/ctan/tex-archive + -- World Wide Web access on http://www.tex.ac.uk/tex-archive + -- Administrator: <ctan-uk@tex.ac.uk> + + tug2.cs.umb.edu (Massachusetts, USA) + -- anonymous ftp /tex-archive (/pub/archive) + -- World Wide Web access on http://tug2.cs.umb.edu/ctan/ + -- Administrator: <ftpmaint@mail.tug.org> + +- -- +Olaf Weber + +------------------------------ + +From: wz@iwd.uni-bremen.de (Roland Weibezahn) +Date: Tue, 18 Nov 1997 10:18:58 +0100 +Subject: ANNOUNCE: xtem version 6 + +Dear xtem users, + +We have finished the new version (xtem_TeXMenu 6.12) of + + "xtem", an X11-TeX-menu + +which runs with the new Tcl/Tk/TclX (Tcl8.0) +as well as with the old versions (Tcl7.4, Tcl7.5 and Tcl7.6). + +In addition to adapting xtem to the new Tcl/Tk, we have done +error corrections and added new features, such as: + +- - the hyphenations found in the .dvi-file can be displayed after a + TeX run (for this we have written a program hyphen_show.c); + +- - we also realized key bindings, + +- - and we modified the print menu (page selection is now controlled + for the existence of the selected page numbers in the document). + +As usual you can get all material (including the source code of +Tcl/Tk/TclX we used) from our ftp server: + + ftp://ftp.iwd.uni-bremen.de/pub/tex/xtem/xtem_texmenu.tar.gz + ftp://ftp.iwd.uni-bremen.de/pub/tcl/*.tar.gz +or + http://ftp.iwd.uni-bremen.de/xtem/xtem_texmenu.html + +The final release took a little bit more time than intended by +us originally (vacancies, installation, and -- last but not least -- +we had hard to work to program around some incompatibilities between +Tcl8.0 and Tcl7.6). + +We have plans for more features, we will discuss them in a few days +in our xtem discussion list, for which you can subscribe with an +e-mail to + majordomo@iwd.uni-bremen.de +with "body" + subscribe xtem-list + end + +Regards, Roland Weibezahn +- -- +Dr. Roland Weibezahn +weibezahn@iwd.uni-bremen.de phone: +49-421-218-3532 +University Bremen, IWD, post-box: 330440, 28334 Bremen, Germany +http://www.iwd.uni-bremen.de/xtem/xtem_texmenu.html (the xtem_TeXMenu project) + +------------------------------ + +From: P Taylor <esrni@eng.warwick.ac.uk> +Date: Wed, 26 Nov 1997 11:26:18 +0000 (GMT) +Subject: Harvard Refs? + +>Hi, +I'm hoping that you can offer advice to me. I have been using Bibtex +C Version 0.99c to obtain standard numerical referencing in the past in +conjunction with LaTeX Version 2.09 (14 Jan 1992). +However, I am just starting to write up my PhD thesis in Civil +Engineering using LaTeX2e (1995/06/01) patch level 3 because +I wish to use the harvard reference system in conjunction with Bibtex +C Version 0.99c. From text books I have managed to learn that I will need the +harvard.sty file written by P Williams and T Schnier. I have obtained +this using ftp however I cannot find out what commands are needed or +where they are needed to get anything working. +I would be very grateful if you could send me the infomation I require or +refer me to a text containing the infomation. Whilst I'm fairly +computer literate I'm no whizz kid (my PhD is in Soil Mechanics!) so I'd +appreciate it if I only new the basics just to get the thing working. +Yours sincerely +Paul Taylor. + +------------------------------ + +From: Werenfried Spit <w.spit@witbo.nl> +Date: Thu, 27 Nov 1997 15:07:29 +0100 +Subject: Harvard Refs? (A) + +There used to be a user manual with harvard.sty. I remember it lived +on CTAN among the bibstyles. +Otherwise, its fairly easy: +- - use a Harvard-bibstyle +- - use normal \cite s +good luck +- --- +Werenfried Spit e-mail W.Spit@witbo.nl +Witteveen+Bos Consulting Engineers phone +31-70-3.700.709 +Den Haag, The Netherlands fax +31-70-3.600.098 + +------------------------------ + +From: Mark Joy <ma_s443@crystal.kingston.ac.uk> +Date: Mon, 15 Dec 1997 10:58:28 GMT0BST +Subject: LaTeX2e version in color.sty + +Can anyone help with the following... + +I have a copy of PCTEX and have copied the installation files for +LATEX 2e from the CTAN site at Aston UK. Everything seems to be ok +except when I try to set up the graphics capabilities. In my +color.sty file I have the command + +\NeedsTeXFormat{LaTeX2e}[1995/12/01] + +generating the warning, + +LaTeX Warning: You have requested release `1995/12/01' of LaTeX, + but only release `1994/12/01' is available. + +whilst typesetting grfguide.tex. I also get the error messages, + +! Illegal parameter number in definition of \ds@monochrome. +<to be read again> + l.50 ...@lor@special\m@ne{define #1 #2}}}} + +etc, etc. + +Thanks in advance +Mark Joy + +------------------------------ + +About TeXhax... + +Please send contributions to: TeXhax@tex.ac.uk + +Subscription and unsubscription requests: + send a one line mail message to TeXhax-Request@tex.ac.uk + containing only the line + subscribe texhax + or + unsubscribe texhax +If you have problems with un/subscribing, +please mail texhax-owner@nottingham.ac.uk + +For information on the TeX Users Group, please send a message to +TUG@mail.tug.org, or write TeX Users Group, 1466 NW Front Avenue, +Suite 3141, Portland, OR 97209-2820 USA (phone: 1 503 223 9994, +fax: 1 503 223 3960). + +Backnumbers of all the digests are stored in the Comprehensive TeX +Archive Network (CTAN) and can be retrieved on the Internet by +anonymous ftp. The hosts comprising CTAN include, among others, + ctan.tug.org (158.121.106.16) -- US + ftp.dante.de (129.69.1.12) -- Germany + ftp.tex.ac.uk (128.232.1.87) -- UK +Please use your nearest server, to keep network load down. +The file /tex-archive/CTAN.sites on each of these hosts gives a +list of other sites which maintain full or partial mirrors of the CTAN. +Alternatively, finger ctan@tug.org for full details. + +TeXhax Digest back issues are filed below /tex-archive/digests/texhax/ +Keyword-In-Context indexes are filed in /tex-archive/digests/indexes/ + +A Hypermail version of TeXhax is also available on the World-Wide Web at URL +http://www.tex.ac.uk/tex-archive/digests/hyper/ + +\bye + +End of TeXhax Digest V1997 #11 +****************************** |